VetPrep Flashcards

1
Q

What tumor type is a cat predisposed to developing at vaccination sites?

A

Fibrosarcoma

How well did you know this?
1
Not at all
2
3
4
5
Perfectly
2
Q

Several beef cows present with a history of decreased appetite and excessive salivation. On physical exam, their tongues are firm on palpation, nodular, and painful

a) what is your top DDX
b) What is your recommendation to the owner?:

1 - Begin therapy with an aminoglycoside

2 - Change feed and treat

3 - Isolate animals at once

4 - Isolate affected animals and submit one of them for necropsy

A

a) actinobacillosis
b) 2 - change feed and treat
explanation: The correct answer is to change feed and begin treatment of the animals affected with woody tongue, as the response is often good. Sodium iodide and antibiotics are effective. Given the presentation and clinical signs these animals have probably begun to ingest very rough and stemmed (scabrous) feed items which have injured their mouths. Upon injury, the normal inhabitant Actinobacillus lignieresii invades the soft tissues and causes the characteristic woody tongue granulomatous inflammation. These animals don’t have rabies, and there is no need to cull them. Change feed before additional animals are affected. Do not use aminoglycosides as they have an extremely long withdrawal period.

How well did you know this?
1
Not at all
2
3
4
5
Perfectly
3
Q

A 13-year old spayed female Miniature Poodle presents with two mobile, soft, well-circumscribed, subcutaneous sternal masses, 3 cm and 6 cm in diameter, respectively. The owner had noticed one mass three months previously. Fine needle aspiration of both masses is performed and smears are prepared. A representative field of both specimens is illustrated below (Wright’s 20X). What is your diagnosis?

  • a) Mesenchymal neoplasia (sarcoma)
  • b) Fungal granuloma
  • c) Lipoma
  • d) Mast cell tumor
A

c) lipoma

Explanation: The cells depicted are well-differentiated adipocytes. These cells are large, appear in aggregates or sometimes singly, and contain fat that stains negatively with Wright’s, such that the cytoplasm appears clear. The cells possess a small, round or ovoid, pyknotic nucleus that may be compressed and located in the periphery of the cell. This benign tumor is a lipoma, which is common in dogs. If the mass hinders the animal, it may be removed surgically. Infiltrative lipomas, and their malignant counterpart, liposarcomas, are less common.

When sampling for cytology, frequently the first indication that a lipoma has been aspirated is the clear, oily appearance of the material ejected from the aspiration needle onto the slide. Care must be taken while staining to ensure that the material does not wash off the slide, as fat does not adhere readily to the glass surface.

Note: Gentle heat fixing of greasy material to a slide may be of benefit in keeping adipose cells adherent to slides during staining. This can be accomplished by holding a slide over a Bunsen burner or lighter or gentle heating on a heating tray/bar for a few seconds so that the side opposite to that containing the cellular material is slowly warmed. The slide must be left to cool completely before staining. Slides coated with poly-l-lysine, which are used for Papanicolaou staining and/or increased adherence of tissue sections to slides, are also helpful in promoting cellular adherence and eliminating loss of cells during staining.

Sometimes, local fat will be aspirated and cannot be reliably differentiated from the adipose cells of a lipoma. If there is any doubt as to the presence of a discrete mass, surgical removal with histological evaluation is recommended.

How well did you know this?
1
Not at all
2
3
4
5
Perfectly
4
Q

A 5-year old indoor/outdoor male neutered short hair presents for a wound on the chest. The owner noticed a hole in the skin and thought he had a ruptured abscess. On close examination of the wound, you notice a larva inside the hole. Which organism is most likely under the skin?

  • a) Ctenocephalides
  • b) Culicoides
  • c) Cuterebra
  • d) Ancylostoma
A

c) cuterebra

Cuterebra is a fly that lays its eggs on soil or plants. The eggs stick to the animal’s fur when they come into contact. The eggs hatch, and the larvae either penetrate the skin, are ingested when the animal grooms, or they enter the animal’s body through a natural opening, such as the nose.

I_n most cases, the larvae migrate to areas just under the skin on the head, neck, or trunk_ of the animal. In dogs, cats, and ferrets, who are not the usual hosts of this parasite, the larvae may also migrate to the brain, eye, eyelids, or throat.

As the larva grows under the skin, it produces a nodule or swelling. A small opening develops in the skin, through which it breathes. A small amount of drainage may occur around this breathing hole.

The treatment is to incise the skin if needed to remove the larva. You have to make sure to remove the larva in whole and not crush it. If it is damaged or crushed, it can cause an anaphylactic reaction.

Ctenocephalides is a flea, Ancylostoma is the hookworm, and Culicoides are tiny gnats that most often bother horses and livestock.

How well did you know this?
1
Not at all
2
3
4
5
Perfectly
5
Q

What is a common side effect of xylazine administration in cats?

  • a) Anuria
  • b) Vomiting
  • c) Seizures
  • d) Polycythemia
  • e) Miosis
A

b) Vomitting

Xylazine frequently causes vomiting in cats. In fact, veterinarians use Xylazine when they wish to induce emesis in cats. It can also cause decreased PCV, mydriasis, and diuresis.

How well did you know this?
1
Not at all
2
3
4
5
Perfectly
6
Q

Once infected, for what period of time is canine parvovirus usually shed?

  • a) 5-6 months
  • b) 1-2 months
  • c) 2-3 months
  • d) 7-10 days
A

d) 7-10 days

The correct answer is 7-10 days. Canine parvovirus mainly affects puppies and young dogs less than a year of age. Transmission of the virus is through contact with infected feces and fomites such as hands, toys, the dog’s hair coat. The virus is very resilient in the environment and is resistant to many types of disinfectants. It replicates in the crypt epithelium of the gut and causes epithelial necrosis and hemorrhagic diarrhea. The virus can also affect the heart of young puppies, causing myocarditis. This occurs less commonly now since most bitches are immunized against the virus, which allows for maternal antibodies to protect young puppies from this form of the disease.

How well did you know this?
1
Not at all
2
3
4
5
Perfectly
7
Q

You go to a ranch to evaluate the sudden death of a bull. There is black, bloody discharge from all orifices. There is incomplete rigor mortis despite being dead for a day. What is your most likely diagnosis?

  • a) Grass staggers (hypomagnésémie, tétanie d’herbage)
  • b) tetanus
  • c) anthrax
  • d) botulism
  • e) Moldy sweet clover toxicity
A

c) Anthrax

Anthrax is caused by Bacillus anthracis. The black blood from the orifices, incomplete rigor mortis, and acute death is very characteristic of the disease in cattle. Lesions occur in the reticuloendothelial system and vasculature. Do not perform a necropsy on the animal as you would potentially release spores into the environment. You should notify the authorities if you suspect the disease.

How well did you know this?
1
Not at all
2
3
4
5
Perfectly
8
Q

Coat color and sun exposure likely predisposed this cat to developing the tumor seen in the photo.

  • a) Basal cell tumor
  • b) Squamous cell carcinoma
  • c) Mast cell tumor
  • d) Melanoma
A

White cats or cats with areas of white fur on the face or ears are predisposed to developing squamous cell carcinoma from UV light. These lesions are usually ulcerative and appear around the nose, ears, or eyelids.

How well did you know this?
1
Not at all
2
3
4
5
Perfectly
9
Q

What is the main mode of transmission for feline leukemia virus?

  • a) Shedding of virus via feces
  • b) Shedding of virus via saliva
  • c) Aerosol transmission
  • d) Fomite transmission
A

b) Shedding of virus via saliva

The main mode of transmission is via saliva. It requires prolonged, close contact. Cats may shed the virus for months to years. Transmission may also occur through reuse of instruments and blood. Virus is shed in saliva, tears, urine, and feces.

How well did you know this?
1
Not at all
2
3
4
5
Perfectly
10
Q

A 6-year old intact male domestic short haired cat presents with acute onset exophthalmos of the right eye in the past 2 days. He is painful on palpation around his eye and head. The eye can be retropulsed some, but there is resistance present. He has not been eating in the past day and has a rectal temperature of 103.4F (39.7 C). What is the most likely diagnosis?

  • a) Glaucoma
  • b) Anterior uveitis
  • c) Orbital neoplasia
  • d) Retrobulbar abscess
A

d) Retrobulbar abscess

Acute, painful exophthalmos is usually due to retrobulbar abscessation. These lesions are usually painful and may be swollen. These animals are frequently systemically ill with fever and leukocytosis. Causes of retrobulbar abscesses include penetrating wounds, foreign bodies, spread from dental or sinus infection, and hematogenous spread. Glaucoma does not cause exophthalmos, although, chronically it can cause buphthalmos which may appear similar. Anterior uveitis also does not cause exophthalmos.

How well did you know this?
1
Not at all
2
3
4
5
Perfectly
11
Q

You are working with a farmer who is having trouble with calves between the age of 2 weeks and 6 months. They are alert, but weak, dyspneic and die suddenly. On necropsy they have pale cardiac and skeletal muscles. What is the farmer’s problem?

  • a) Sorghum toxicity
  • b) Copper deficiency
  • c) Selenium deficiency
  • d) Lightning strike
A

c) Selenium deficiency

The pale muscle and clinical signs are classic for vitamin E and selenium deficiency. This is important to remember! Other things that should be on your differential list for this case include cardiotoxic plants.

How well did you know this?
1
Not at all
2
3
4
5
Perfectly
12
Q

A 5-month old female cat presents to you for weight loss, chronic diarrhea and steatorrhea. The organism seen in a stained fecal smear is shown in the image below (this is a magnified 40X image, the organism is approximately 15 x 8 um). Which treatment is most appropriate

  • a) Sulfadimethoxine
  • b) Tylosin
  • c) Penicillin
  • d) Fenbendazole
A

d) Fenbendazole

The organism is Giardia which can be recognized as a trophozoite with two nuclei outlined by adhesive discs. Giardia should be distinguishable from trichomonads which have a single nucleus and an undulating membrane.

The best treatments for Giardia are either fenbendazole or metronidazole.

How well did you know this?
1
Not at all
2
3
4
5
Perfectly
13
Q

A 10-year old female spayed Siamese cat presents for a new lump the owner found a month ago. She was spayed last year before she was adopted from the shelter. The owner states that the lump has grown over the last month, and it doesn’t seem to bother the cat. On exam, the 2 cm lump is located on the left 2nd mammary gland, and no other lumps are noted. The lump is freely moveable, and chest radiographs are clear. What is the chance that this tumor is malignant?

  • a) 50%
  • b) 85%
  • c) 5%
  • d) 20%
A

b) 85%

70-90% of all feline mammary tumors are malignant.

Spaying a cat before 6 months of age reduces the risk for mammary tumors by 7 fold.

The statistics for dogs is a 50:50 chance of malignancy for a single mammary mass.

How well did you know this?
1
Not at all
2
3
4
5
Perfectly
14
Q

An 8-week old Abyssinian cat recently obtained from a cattery presents to you for an examination and the owner reports that the cat has had diarrhea. On fecal float, you find multiple structures like the one shown in the photo (see image). What should you treat the cat with?

  • a) Praziquantel (Droncit)
  • b) Metronidazole (Flagyl)
  • c) Sulfadimethoxine (Albon)
  • d) Pyrantel (Strongid)
  • e) Selamectin (Revolution)
  • f) Amoxicillin and clavulanate (Clavamox)
A

c) Sulfadimethoxine

This is an image of Isospora from a cat. Isospora are parasitic coccidia that can cause diarrhea as this cat is showing. Treatment for coccidia is usually with sulfonamides such as sulfadimethoxine or trimethoprim sulfa.

For the other drugs listed:

Droncit- Primarily for cestodes (tapeworms)
Revolution- For fleas, heartworms, hookworms, roundworms, and ear mites
Strongid- Primarily for roundworms and hookworms
Clavamox- A broad spectrum antibacterial
Metronidazole- Primarily for anaerobes, also used for giardia

How well did you know this?
1
Not at all
2
3
4
5
Perfectly
15
Q

A 12-year old male neutered domestic short hair cat presents for ongoing evaluation of diabetes mellitus. The cat was diagnosed 6 months ago and has continued to be markedly polyuric, polydipsic, polyphagic, and has been gaining weight. The cat is currently receiving 10 units of glargine insulin every 12 hours. On physical exam, the cat weighs 15 pounds (6.8 kg) and has an enlarged head, abdomen, and paws. What imaging modality would be most appropriate to try and prove what you suspect is causing the uncontrolled diabetes and weight gain in this cat?

a) Ultrasound of the neck

  • a) Ultrasound of the neck
  • b) Ultrasound of the abdomen
  • c) MRI of the head
  • d) Radiographs of the thorax
  • e) CT of the abdomen
A

C) MRI of the head

This cat has the signs and symptoms of acromegaly. Acromegaly is caused by excessive growth hormone release from the pars distalis from a tumor in the pituitary gland. Excessive growth hormone causes a defect in the insulin receptors on target cells causing insulin resistant diabetes mellitus. The enlarged head, paws, abdomen, and weight gain despite uncontrolled diabetes is due to the anabolic effects of the growth hormone.

Treatment for this condition includes radiation therapy to the pituitary tumor, high doses of insulin to try and control the diabetes, and somatostatin* analogs (octreotide) to try and inhibit the release of growth hormone from the tumor. Surgical excision has been used as a form of treatment in people with pituitary tumors, but this has only been rarely reported in cats.

note (wiki): About 98% of cases of acromegaly are due to the overproduction of growth hormone by a benign tumor of the pituitary gland called an adenoma.[16] These tumors produce excessive growth hormone and compress surrounding brain tissues as they grow larger. In some cases, they may compress the optic nerves.

* Somatostatin = Growth Hormone Inhibiting Hormone, GHIH

How well did you know this?
1
Not at all
2
3
4
5
Perfectly
16
Q

A 36 hour foal born from a multiparous mare presents for weakness, decreased appetite, and tachycardia. On physical exam, you examine the sclera as seen in the photo. Based on the signalment and findings, what is a likely cause?

  • a) Congenital iron toxicity
  • b) Neonatal Isoerythrolysis
  • c) Immune mediated thrombocytopenia
  • d) Failure of passive transfer
A

B) Neonatal Isoerythrolysis

Neonatal Isoerythrolysis (NI) would be a possible cause of the icterus noted in the sclera. NI is common in multiparous mares and usually affects foals between 24-72 hours of age. NI results in hemolysis, hyperbilirubinemia, and icterus. A low packed cell volume would further support NI.

NI develops because the newborn foal expresses alloantigens on its red blood cells inherited from the sire that the mare does not have. If the mare becomes sensitized to the sire’s specific antigen, maternal antibodies are produced and absorbed by the foal soon after birth. This subsequently results in lysis of the red blood cells.

How well did you know this?
1
Not at all
2
3
4
5
Perfectly
17
Q

You examine a very ill 4-year old Holstein dairy cow on a large commercial dairy. She freshened one week ago and was producing well, until she was found down and unwilling to rise this morning when you were called. T=103F or 39.4 C, HR=90, and RR=35. The scleral vessels are dark are enlarged, her rumen is fairly empty and the motility is poor, and she appears too weak to rise. Rectal exam reveals an involuting uterus which can be retracted, discharging a brownish red mucoid non-odorous lochia through the vagina. The left rear quarter of her udder is swollen, hot, painful, and discolored (see image), and contains a serum-like secretion with clumps of fibrin in it. What is your diagnosis?

  • a) Hypocalcemia (milk fever)
  • b) Coliform mastitis
  • c) Displaced abomasum
  • d) Grain overload
  • e) Metritis
A

B) Coliform mastitis

This is a case of severe acute coliform mastitis, and the absorbed endotoxin (LPS) is causing many of the systemic signs observed. The cow needs to be aggressively treated with IV fluids, NSAIDS, and supportive nursing. The gland should be frequently milked out. The use of both intramammary and systemic antimicrobial drugs to which most coliforms are susceptible is still controversial, but is often done in cows in a severe state of illness as in this case. While this cow may have secondary hypocalcemia, treatment with calcium needs to be approached cautiously, as endotoxic animals have very sensitive myocardium and arrest may occur if calcium is given IV. If given, preferred routes of calcium administration would be subcutaneous or oral.

How well did you know this?
1
Not at all
2
3
4
5
Perfectly
18
Q

A 2-year old female spayed Dachshund presents for further evaluation as a result of a mass on the pinna noted by the owner. On physical exam, the only abnormality noted was this mass. Cytology is consistent with a histiocytoma. Which of the following is the most reasonable treatment option?

  • a) Benign Neglect
  • b) Radical surgical excision
  • c) Cryotherapy
  • d) chemotherapy using lomustine
  • e) Radiation therapy
A

A

Histiocytomas are typically benign and will often present as a small, raised mass that may or may not be ulcerated. Fortunately, these masses usually regress on their own, and surgical or medical intervention is typically not necessary.

How well did you know this?
1
Not at all
2
3
4
5
Perfectly
19
Q

A 4-year old female ferret presents with a history of an enlarged vulva and truncal alopecia. What is the most likely diagnosis?

  • a) Adrenal tumor
  • b) Hypothyroidism
  • c) Sertoli cell tumor
  • d) Insulinoma
A

A

The correct answer is adrenal tumor. These tumors are very common in middle-aged ferrets and usually secrete estradiol. Common clinical signs are hair loss, enlarged vulva, pruritus, and behavior changes. Diagnosis is made by clinical signs, ultrasound, and measurement of serum estradiol levels. Treatment is adrenalectomy.

How well did you know this?
1
Not at all
2
3
4
5
Perfectly
20
Q

What is the primary mode of transmission of West Nile Virus (WNV) between birds?

  • a) Via Cnemidocoptes mites
  • b) Via mosquitoes
  • c) Via feces
  • d) Via placenta
  • e) Via saliva
A

B

Mosquito vectors are the primary mode of transmission for WNV between birds and other hosts (horses, humans). There are infrequent documented cases of the disease being spread by feces or saliva. No mites have been documented to transmit the disease. Birds do not have a placenta.

How well did you know this?
1
Not at all
2
3
4
5
Perfectly
21
Q

A 7-year old female spayed English Springer Spaniel presents for difficulty breathing and lethargy. Physical exam shows pale mucous membranes and increased respiratory effort. You collect blood for a complete blood count and a chemistry panel and you place a drop of blood with a drop of saline on a glass slide. After swirling the sample around, you can grossly see what is shown in the picture. What’s your diagnosis?

  • a) Hemolysis
  • b) Immune mediated hemolytic anemia
  • c) Evan’s syndrome
  • d) Vitamin K antagonist toxicity
  • e) Rouleaux formations
A

B) Immune Mediated Hemolytic Anemia

The image shows a positive slide agglutination test. The dark patches in the sample are caused by antibodies attached to the surface of the erythrocytes cross linking the cells together causing these clumps. This is highly suggestive of an immune mediated reaction to antigen on the individual’s red blood cells.

Rouleaux formations are red blood cells stacked together as a result of their natural discoid shape and large surface area causing them to have an affinity for each other. They typically resemble a stack of coins or cookies.

Evan’s syndrome is an autoimmune condition characterized by an immune attack on erythrocytes, platelets, and occasionally leukocytes as well. You w_ould not be able to diagnose this on a slide agglutination test alone_.

Vitamin K antagonist toxicity with rodenticide causes a coagulopathy but should not cause hemagglutination.

How well did you know this?
1
Not at all
2
3
4
5
Perfectly
22
Q

A 6-month old Chocolate Labrador presents for limping and failure to gain weight. He is housed in an outdoor kennel with other hunting dogs. He is fed a large breed dry puppy food. He received his puppy shots at 8, 12, and 16 weeks old. He is quiet, alert, and responsive. His body condition score is 3/9, with rib exposure and poor fat deposition. His mucous membranes are pale pink, with a capillary refill time of 2 seconds. His heart and lungs auscult normally. No abnormalities are felt on abdominal palpation. He is an intact male, and both testicles are descended. The only abnormalities are the pads of his two front feet and left hind foot (see image) that he chews at frequently. What is diagnostic test of choice?

  • A) Skin scraping
  • B) Biopsy
  • C) Radiograph
  • D) CBC and chemistry panel
  • E) fecal float
A

E) Fecal Float

Hookworms (Ancylostoma and Uncinaria) are intestinal parasites that suck blood and can cause anemia, enteritis, coughing during larval migration, and dermatitis. Any young dog that is failing to thrive and/or has pale mucous membranes should be tested for intestinal parasites. Hookworm dermatitis, also called Ancylostomiasis is typically seen in conditions with poor sanitation and/or in kennels.

Hookworms can be transmitted in utero, during nursing, or via 3rd stage larva penetrating the skin. The most commonly affected skin areas are the pads and interdigital spaces of the feet, but can include any surface that contacts the ground. The larva migrate through the dog’s tissues before arriving in the intestines. They cause significant anemia, failure to thrive, or sudden death in young dogs. In mild cases, deworming protocols are often enough; with severe cases blood transfusions and parenteral treatments are often necessary.

Skin scraping and impression smears of the affected skin areas are typically unrewarding for isolating parasites. A PCV/TP would show signs of anemia, but not the underlying etiology. Complete blood cell count and chemistry will often show anemia that is regenerative and an eosinophilia. Radiographs would be unrewarding in this case. A biopsy may show migrating larva if biopsied soon after trauma, however this is not a diagnostic test routinely used.

How well did you know this?
1
Not at all
2
3
4
5
Perfectly
23
Q

A 2-year old mare presents to you several weeks after recovering from a mild upper respiratory infection. She now presents with edema and sloughing of the legs (see image), chest and abdomen as well as mucosal petechial hemorrhages. She is sore and reluctant to move. Biopsy of the skin lesion is consistent with aseptic necrotizing vasculitis. What is the most likely diagnosis?

  • a) Type I Hypersensitivity
  • b) Bastard Strangles
  • c) Purpura hemorrhagica
  • d) Idiopathic thrombocytopenia
A

C) Purpura hemorrhagica

This is the clinical and histologic appearance of purpura hemorrhagica. It is a type-III hypersensitivity which is when antigen-antibody complexes accumulate, leading to disease.

Purpura hemorrhagica most commonly occurs 2-4 weeks after exposure to certain infectious agents or vaccines. This condition is most commonly seen subsequent to infection with Streptococcus equi subsp. equi or vaccination against it but it can also be associated with other pathogens, particularly respiratory pathogens including other streptococcal species and equine influenza.

Regardless of the cause, purpura hemorrhagica results from accumulation of antigen-antibody complexes that deposit on blood vessel walls and activate a strong immune response (vasculitis). The leaky blood vessels lead to hemorrhage and edema.

Bastard strangles refers to the condition when Streptococcus equi subsp. equi creates abscesses in unusual sites (other than the lymph nodes draining the throat) such as abdominal or lung lymph nodes.

Cantharidin toxicity (also known as blister beetle toxicity) leads to mucosal irritation and results in colic and cystitis. It can also lead to hypocalcemia.

Primary immune-mediated thrombocytopenia (sometimes called idiopathic thrombocytopenia) can be seen in horses but is not consistent with the necrotizing vasculitis described in the case.

Type I hypersensitivity reactions are immediate reactions (such as urticaria) invoked by an antigen or allergen.

How well did you know this?
1
Not at all
2
3
4
5
Perfectly
24
Q

A 2-year old nonlactating dairy cow in California has a sudden onset of head tilt and drooling. The owner also reports that she appears less active and less interested in feed than normal, and today is circling in one direction in the pen. You examine her and find T=105F (40.6 C), HR=96, and RR=32. There is ptosis, drooped ear, and weakness of the lips on the affected side. You take a lumbosacral spinal tap (see photo), and submit the CSF to your lab. The results show elevated protein and WBCs, with the cell type being mainly monocytes. The lab reports seeing some gram positive bacteria in the monocytes of the CSF. Based on these findings what is the best treatment for this condition?

  • A) Enrofloxacin
  • B) Penicillin
  • C) Metronidazole
  • D) Metoclopramide
  • E) Chloramphenicol
A

B) Penicillin

The diagnosis is Listeriosis. Listeria monocytogenes can effectively be treated in the early stages of the disease with penicillin, ampicillin, or tetracycline. Intramuscular procaine penicillin for example has a withdrawal of 10 days for slaughter (meat) and 48 hours for milk if the animal is lactating. Other choices of approved antimicrobials would also likely be effective since L. monocytogenes is susceptible to most antimicrobials. For a list of approved animal drugs see www.farad.org/vetgram. FARAD is the United States Food Animal Residue Avoidance Databank which gives withdrawal information as well as a list of prohibited drugs. The other drugs listed as choices here are prohibited and cannot be legally used in food animals in the USA.

How well did you know this?
1
Not at all
2
3
4
5
Perfectly
25
Q

A dog presents to your clinic with tenesmus and swelling near the anus as seen in the image. On examination, there is a fluctuant swelling lateral to the anus, and on rectal exam, you note lateral dilatation of the rectum. Which of the following is the most common signalment for dogs presenting with this problem?

  • A) They are younger intact females
  • B) They are younger intact males
  • C) They are older intact females
  • D) They are older intact males
  • E) There is no age or sex predilection
A

D) They are older intact males

This is a case of a perineal hernia. Older intact male dogs are most commonly affected.

Overrepresented breeds include Boxers, Collies, Kelpies, Pekingese, and Boston terriers. The hernia results from a weakened pelvic diaphragm. It is thought that there may be a hormonal component which results in weakening with time.

Wiki:

Perineal hernia is a hernia involving the perineum (pelvic floor). The hernia may contain fluid, fat, any part of the intestine, the rectum, or the bladder.

Dogs with benign prostatic hyperplasia have been found to have increased relaxin levels and suspected subsequent weakening of the pelvic diaphragm

In cats, perineal hernias are seen most commonly following perineal urethrostomy surgery or secondary to megacolon.

How well did you know this?
1
Not at all
2
3
4
5
Perfectly
26
Q

A pregnant mare was brought out to your barn for observation in anticipation of parturition. After several hours of restless behavior, several gallons of allantoic fluid rush out from the vulva. Which of the following would you expect to happen next for a normal parturition?

  • A) The thin, white, glistening amniotic membrane emerges from the vulva
  • B) The red, velvety, chorioallantoic membrane emerges from the vulva
  • C) The placenta is expelled from the vulva
  • D) The hind legs of the foal emerge from the vulva
A

A) The thin, white, glistening amniotic membrane emerges from the vulva

This case description is consistent with stage I of labor in the horse. The first stage of foaling typically lasts 30 minutes to 4 hours. During this stage, mares act restless and may exhibit signs similar to colic such as flank watching, pawing, and constantly getting up and down. When the placenta ruptures (“water breaks”), there may be several gallons of allantoic fluid that come out. Usually, within about 5 minutes, the second stage of labor begins and the foals feet and nose appear at the vulva, covered in the white, thin, glistening amnion. If a red, velvety, membrane is seen, this is the chorioallantois which indicates premature placental separation which can impair oxygen delivery to the fetus and can result in death of the foal. Usually, the muzzle will emerge from the amnion by the time the foal’s hips pass through the pelvis but if not, the amnion can be gently broken and removed. Usually, the umbilical cord breaks naturally when the mare stands or foal begins to rise. Then, within 30 minutes to 3 hours after foaling, the placenta should be expelled.

How well did you know this?
1
Not at all
2
3
4
5
Perfectly
27
Q

A 2-year old male neutered cat presents to you depressed, hypersalivating, and ataxic with muscle tremors. The owner reports that a pyrethrin-based spot-on formulation for flea control belonging to their Golden Retriever was accidentally applied on the cat earlier today. Which of the following drugs will you use to treat the cat’s clinical signs?

  • A) Methocarbamol
  • B) Amoxicillin
  • C) Acepromazine
  • D) 2-PAM
  • E) Atropine
A

A) Methocarbamol

Pyrethrins alter the activity of the sodium ion channels of nerves, which prolongs the period of sodium conductance. This increases the length of depolarization resulting in repetitive nerve firing. Cats are particularly sensitive to pyrethrin-containing products and can develop clinical signs within hours after administration. Affected animals should be bathed to remove remaining product. Minor clinical signs such as hypersalivation and ear twitching are usually self-limiting and do not require treatment. Control of marked tremors or seizures can be achieved with methocarbamol (Robaxin).

How well did you know this?
1
Not at all
2
3
4
5
Perfectly
28
Q

Your client is pregnant and is worried about acquiring toxoplasmosis from her cat. What do you advise?

  • A) Have a housemate empty the litter box daily as a simple precaution to prevent infection as it takes 1-3 days for passed oocysts in the stool to sporulate into an infective form.
  • B) Toxoplasmosis titer should be performed on the owner by a human physician. A positive titer indicates antibodies to the organism that will prevent infection in the first trimester. A negative titer indicates she should remove her cats from her environment.
  • C) Submit toxoplasmosis titers from the cat. A IgG titer of 1:64 or greater suggests recent or active infection that could pose a danger.
  • D) Submit toxoplasmosis titers from the cat. A positive cat infected with toxoplasmosis can shed multiple times in its lifetime and pose a zoonotic risk.
A

A

Toxoplasmosis gondii is a protozoal organism. The cat is the definitive host; the entire life cycle of the organism can be completed within this host. Most cats become infected when they consume an exposed rodent with bradyzoites encysted in their tissues. Only recently infected cats generally shed oocysts in their stool, and cats typically only shed these oocysts for 1-2 weeks. Most cats will only have one shedding episode in their lifetime.

A IgM (not IgG) titer of 1:64 or greater suggests recent or active infection and that cat is at risk of shedding oocysts in their stools. Oocysts are not infective until they sporulate. This process takes > 24 hours, so emptying the litter box daily is advised, preferably by someone who is not pregnant.

If an owner has owned cats for a long while, it is possible that they may have previously been exposed and therefore have mounted an immune response to the organism. If so, it may be advisable to test for Toxoplasma antibody titers in the owner. A sufficient antibody titer will mean the client is protected from infection during the first trimester.

How well did you know this?
1
Not at all
2
3
4
5
Perfectly
29
Q

What is the potential udder fate of goats infected with caprine arthritis encephalomyelitis virus?

  • A) Gangrene
  • B) The udder is not affected
  • C) Hardbag
  • D) Bluebag
A

C

CAEV is a retrovirus and when it affects the udder it will cause fibrosis and result in a firm udder with agalactia. Treatment is ineffective and the goat should be culled. The disease is usually subclinical but can cause arthritis in adults and encephalitis in kids.

How well did you know this?
1
Not at all
2
3
4
5
Perfectly
30
Q

A 4-year old male Manx cat presents to you because the owners found an empty, opened pill vial in the bathroom and the cat vomited. On physical exam, you note ptyalism and facial edema. The cat’s mucous membranes are pale and slightly icteric. You perform a blood smear and detect Heinz bodies in erythrocytes. The cat’s packed cell volume (PCV) is 26% (30-45%). The owners provide you a list of the medications in the medicine cabinet which are acetaminophen (Tylenol), finasteride (Propecia), enalapril (Vasotec), and omeprazole (Prilosec). What treatments should you institute for this cat?

  • A) Activated charcoal and whole blood transfusion
  • B) Prednisone and amoxicillin
  • C) Acetylcysteine and S-adenosylmethionine
  • D) Emesis and methylene blue
A

C) Acetylcysteine and S-adenosylmethionine

Acetaminophen toxicity in cats usually occurs when owners administer the drug, unaware of its significant potential toxicity in cats. In this case, the cat’s clinical signs are most consistent with acetaminophen toxicity based on the Heinz body anemia that is present. Cats can die from oxidative damage and methemoglobinemia within 1-2 days of ingestion. It may also be associated with hepatotoxicity in cats, although this is seen more frequently in dogs.

Recall that cats are particularly sensitive to acetaminophen because they have decreased glucuronyl transferase activity which conjugates acetaminophen to glucuronic acid for excretion. As a result, 50-60 mg (a single tablet) may be fatal for a 4-5 kg cat.

Treatment should consist of toxin removal if possible by inducing emesis in some cases. As the cat in this case is already vomiting, this may not be necessary. Activated charcoal is controversial and should only be given if ingestion occurred within hours and should be administered very carefully in cats due to the risk of aspiration.

The specific antidote is acetylcysteine which binds to some of the reactive metabolites of acetaminophen and increases the availability and synthesis of glutathione. Other treatments may include S-Adenosylmethionine (SAMe) which has hepatoprotective and antioxidant properties. Cimetidine can be given to inhibit the p450 oxidase in the liver and limit formation of toxic metabolites. Ascorbic acid can also be used as an adjunct treatment to bind toxic metabolites. In cats with signs of hypoxemia from severe hemolytic anemia (PCV <20%), a transfusion and further supportive care may be warranted.

How well did you know this?
1
Not at all
2
3
4
5
Perfectly
31
Q

A client calls and says her cat was chewing on her lily plant two days ago and is now acting very sick. You tell her to bring the cat in immediately so that you can perform which of the following?

  • a) ALT, AST, GGT, and total bilirubin levels
  • b) Induce emesis and administer activated charcoal
  • c) BUN and creatinine levels
  • d) Abdominal radiographs to diagnose intestinal obstruction
A

C) BUN and creatinine levels

Lily plant toxicosis is extremely serious and can cause rapid and fatal acute renal failure in cats. If ingestion is suspected, decontamination and aggressive fluid therapy, and monitoring of renal values are recommended immediately.

In this case, since 2 days have passed, inducing emesis and administering activated charcoal would not be helpful. What you can do is check renal values (BUN and creatinine) and treat for acute renal failure.

Ruling out an intestinal obstruction in a cat with this history is reasonable but not the best of the choices given.

Web: Lilies in the “true lily” and “daylily” families are very dangerous for cats. The entire lily plant is toxic: the stem, leaves, flowers, pollen, and even the water in a vase. Eating just a small amount of a leaf or flower petal, licking a few pollen grains off its fur while grooming, or drinking the water from the vase can cause your cat to develop fatal kidney failure in less than 3 days. The toxin, which only affects cats, has not been identified. Dogs that eat lilies may have minor stomach upset but they don’t develop kidney failure.

How well did you know this?
1
Not at all
2
3
4
5
Perfectly
32
Q

A 3-year old, indoor-only, male castrated Maine Coon cat is presented to you for acute-onset of hindlimb pain and paresis. Physical examination reveals a grade III left parasternal heart murmur and minimal movement in the hind legs. Femoral pulses are bilaterally absent, and the hindlimb toes are cold to the touch. The cat is tachypenic, but lung sounds are normal. What is the most likely diagnosis?

  • A) Chlorpyrifos toxicity
  • B) Tetanus
  • C) T3-L3 intervertebral disc disease
  • D) Lymphoma of the spinal cord
  • E) Thrombus at the aortic bifurcation (saddle thrombus)
A

E) Thrombus at the aortic bifurcation (saddle thrombus)

Maine Coon cats are predisposed to development of hypertrophic cardiomyopathy at a young age. Consequently, left atrial enlargement predisposes to atrial thrombus formation, and these clots frequently lodge in the arterial supply to the hindlimbs. The trifurcation is where the aorta divides into the two external iliac arteries and the common origin of the internal iliac arteries. Classic findings due to a clot at the aortic trifurcation include posterior paresis/paralysis, hindlimb pain, cyanotic nailbeds, absent femoral pulses, and a firm leg musculature. Other signs of cardiac disease/failure (murmur or pulmonary edema) are often but not always evident at presentation.

Neither cord lesions, tetanus, nor toxicities should cause the vascular compromise evident on this cat’s physical exam.

How well did you know this?
1
Not at all
2
3
4
5
Perfectly
33
Q

A 4 year old mare has bilateral swelling and drainage of the mandibular lymph nodes. Rectal temperature is 101.8F (38.8 C). Which of the following is the most appropriate plan for this horse?

  • A) Culture the discharge for bacteria
  • B) Administer corticosteroids
  • C) Administer penicillin
  • D) Inform the state veterinarian
A

A) Culture the discharge for bacteria

The correct answer is to culture the discharge for bacteria. The most likely diagnosis for this horse is equine strangles. Strangles most commonly affects younger horses (<5 years of age), but can cause disease in any age horse. The etiologic agent of this disease is S_treptococcus equi subsp. equi_. The diagnostic test of choice to confirm this is bacterial culture. While awaiting culture results, the horse should be separated from any other horses, as strangles is highly contagious to other horses. Antibiotic therapy is controversial and thought to lengthen the course of disease rather than shorten it when given at this stage; also, it may possibly interfere with the natural immunity acquired from natural infection. S. equi infection is not reportable in all states (or countries), and so it may not need to be reported universally.

How well did you know this?
1
Not at all
2
3
4
5
Perfectly
34
Q

A 10 year male castrated German Shepherd dog presents for an acute onset of ptosis, drooping of the lip, and drooping of the cheek all on the left side. What nerve has been damaged?

A

CN VII.

This is the facial nerve which is responsible for motor of the facial muscles as well as sensation in the ear and lacrimation. CN V is mostly responsible for sensation (and motion/tone of the mastication muscles: masseter and temporal muscles). CN III is the oculomotor nerve and serves in movement of the eye. CN VI is also involved in movement of the eye.

Rappel:

  • NCVII : symétrie faciale (oreilles, ouvertures palpébrales*, narines, lèvres, vibrisses). ,
    • ​* aussi testé avec réflexe palpébraux (NCV) et réponse à la menace (NCII)
  • NCV: sensation au visage (réflexes palpébraux + stimulation du septum nasal) et muscles de la mastication (masséter, temporal)
How well did you know this?
1
Not at all
2
3
4
5
Perfectly
35
Q

If a mass appears in the lungs on a right lateral radiograph but not on a left lateral radiograph, where is the mass located?

a) Right lung
b) Left lung
c) mediastinum
d) body wall

A

b) Left lung

The correct answer is left lung. On a right lateral radiograph, the right side is down. In this situation, the right lung lobes are compressed and the left lung lobes inflated, accentuating a mass in the left lung. On a left lateral radiograph, the left lobes are compressed and a mass in the left lung can be concealed due to compression of the lung surrounding the mass.

How well did you know this?
1
Not at all
2
3
4
5
Perfectly
36
Q

A 1-year old female spayed Doberman Pinscher has presented after being hit by a car. Initial chest radiographs show mild contusions, and the patient appears to be otherwise stable. A right mid-shaft long oblique femoral fracture has been identified. Routine pre-operative blood work is unremarkable. A buccal mucosal bleeding test (BMBT) is elevated at 6 minutes. What will you administer prior to surgery?

  • a) Vitamin K
  • b) Whole blood transfusion
  • c) 1,25 dihydrocholecalciferol
  • d) Desmopressin acetate
A

d) Desmopressin acetate

The correct answer is desmopressin acetate (DDAVP). Administration of desmopressin results in release of von Willebrand factor, which will help this patient with clotting. Given this dog’s breed and elevated BMBT there is a very strong likelihood she is afflicted with von Willebrand’s disease. In Dobermans this results in an inability to form a clot. This can be life threatening if the dog is taken to surgery.

A whole blood transfusion does not provide an adequate source of von Willebrand factor but may be necessary if the patient’s bleeding cannot be controlled despite appropriate pre-operative measures. 1,25 dihydrocholecalciferol is the active form of vitamin D which aids intestinal resorption of calcium. The BMBT does not assess factors 2, 7, 9, or 10 and therefore vitamin K is not indicated.

moi: test de saignement = hémostase primaire (formation du clou plaquettaire, impliquant facteur de VwB) et non pas secondaire (coagulation/consolidation du clou)

How well did you know this?
1
Not at all
2
3
4
5
Perfectly
37
Q

The goose shown in the image below was found weak on the shore of a local pond where oil had been dumped. The goose was covered in oil, dehydrated and weak but responsive. Which of the following is an important acute clinical effect of oil on affected birds?

  • a) Disruption of function of the plumage
  • b) Nephrotoxicity
  • c) Hepatotoxicity
  • d) Lead toxicity
A

a) Disruption of function of plumage

Feathers serve a critical waterproofing and insulatory function which is disrupted by oil and can rapidly result in hypothermia. Other concerns for oiled birds include GI irritation from ingestion of oil during preening, hemolytic anemia, and pneumonia due to inhalation of oil.

Treatments include heat, supportive care, and activated charcoal. Once stabilized, frequent high pressure, warm, mild detergent baths and clean warm water rinses until water beads freely off of the feathers is important. Birds should be placed in warm air flow until dry and they should be maintained on self-skimming ponds for several days after washing to ensure full waterproofing.

How well did you know this?
1
Not at all
2
3
4
5
Perfectly
38
Q

Oral administration of which of these drugs has been implicated as a cause of esophageal strictures in cats?

  • a) Potassium bromide
  • b) Doxycycline
  • c) Diazepam
  • d) Azithromycin
A

b) Doxycycline

There is also evidence that clindamycin can cause stricture formation. For this reason, it is recommended that after pilling a cat with doxycycline, it is followed with a small volume (5-10 mls) of water.

How well did you know this?
1
Not at all
2
3
4
5
Perfectly
39
Q

Which infectious agent causes the clinical presentation of pigs most similar to Haemophilus parasuis (Glasser’s disease)?

  • a) Streptococcus suis
  • b) Fusobacterium necrophorum
  • c) Mycoplasma hyosynoviae
  • d) Erysipelothrix rhusiopathiae
A

a) Streptococcus suis.

Streptococcus suis and H. parasuis both cause polyarthritis, polyserositis, fever, and pneumonia in young piglets up to several weeks of age. Both can cause fibrinopurulent inflammation as well as meningitis and convulsions.

Erysipelas and M. hyosynoviae typically occur in grower and finisher pigs and do not result in pneumonia. Diamond-shaped skin lesions (thus the name “diamond skin disease”) are pathognomonic for Erysipelas. Fusobacterium necrophorum causes lameness via footrot or laminitis.

How well did you know this?
1
Not at all
2
3
4
5
Perfectly
40
Q

You are called one hot summer day to see a group of horses in New Mexico which are slobbering and not eating their hay. Three out of 20 horses seem to be visibly affected. On physical exam of the first one, you find fever of 104 F (40 C) and obvious oral ulcers as shown in the image, mainly on the tongue. What is your tentative diagnosis?

  • a) Glanders
  • b) Foot-and-Mouth disease (FMD)
  • c) Bovine papular stomatitis
  • d) Vesicular stomatitis
  • e) African Horse Sickness
A

d. Vesicular stomitis

VS affects horses, cattle and pigs with similar clinical signs. It is a viral disease of high morbidity and low mortality that appears about every 7 to 10 years in the Southwestern United States. Yearly outbreaks occur in southern Mexico and northern South America. Black flies and midges appear to be the vectors. Affected premises should be quarantined.

How well did you know this?
1
Not at all
2
3
4
5
Perfectly
41
Q

In January you examine a group of dairy calves which range in age from 2 to 7 months, with a complaint of hair loss and pruritus. The calves are thin and mucous membranes are pale. One has developed bronchopneumonia, and is also febrile and depressed. You do a skin scraping and find the parasite shown in the image, which your technician identifies as Solenopotes sp. The CBC shows the calves to be severely anemic. What treatment recommendation should you now make to the dairy owner?

  • a) Vaccinate all calves against Mannheimia hemolytica
  • b) Treat all calves for lice
  • c) Treat all calves with hematinics
  • d) Treat all calves for mange
  • e) Treat all calves with long acting tetracycline
A

b) Treat all calves for lice

This is a blood sucking genus of louse, and c_an cause severe anemia_. The anemic calves become thin and m_ore susceptible to diseases like pneumonia_. You can tell this is a louse and not a tick or a mite because lice are insects with 6 legs and ticks and mites are arachnids with 8 legs.
The three genera of blood sucking cattle lice are Solenopotes, Linognathus, and Hematopinus.

How well did you know this?
1
Not at all
2
3
4
5
Perfectly
42
Q

An 8-week old Abyssinian cat recently obtained from a cattery presents to you for an examination and the owner reports that the cat has had diarrhea. On fecal float, you find multiple structures like the one shown in the photo (see image). What should you treat the cat with?

  • a) Sulfadimethoxine (Albon)
  • b) Amoxicillin and clavulanate (Clavamox)
  • c) Praziquantel (Droncit)
  • d) Selamectin (Revolution)
  • e) Pyrantel (Strongid)
  • f) Metronidazole (Flagyl)
A

a) Sulfadimethoxine (Albon)

This is an image of Isospora from a cat. Isospora are parasitic coccidia that can cause diarrhea as this cat is showing. Treatment for coccidia is usually with sulfonamides such as sulfadimethoxine or trimethoprim sulfa.

For the other drugs listed:

Droncit- Primarily for cestodes (tapeworms)
Revolution- For fleas, heartworms, hookworms, roundworms, and ear mites
Strongid- Primarily for roundworms and hookworms
Clavamox- A broad spectrum antibacterial
Metronidazole- Primarily for anaerobes, also used for giardia

How well did you know this?
1
Not at all
2
3
4
5
Perfectly
43
Q

You see an 8-month old kitten with the effusive form of feline infectious peritonitis and perform euthanasia. The kitten was having severe diarrhea around the house when it became ill. The owner has a 2 year old cat at home and wants to know what this cat’s prognosis is since it has been exposed to the sick kitten. Currently this cat is clinically healthy. What do you tell her?

  • a) Perform a PCR on the cat’s feces to see if the virus is being shed
  • b) Her other cat may develop symptoms within the next two weeks because FIP is highly contagious
  • c) Feline infectious peritonitis is not contagious and because her other cat died of FIP does not mean this cat will succumb to the disease
  • d) You recommend a coronavirus titer to determine it the cat is actively infected
  • e) Place the cat on L-lysine to prevent or suppress any infection with FIP
A

C)

Feline infectious peritonitis is not a contagious disease (mais transmission indirecte par voie feco-orale possible non?). It is a disease that is caused by a mutation of feline enteric coronavirus. It is unknown why in some patients this virus mutates and causes the FIP syndrome. It is most likely to occur in young or immunocompromised cats. Her other cat is not necessarily going to get FIP just from exposure. In fact, the majority of the cat population has been exposed to the feline enteric coronavirus.

Because most cats in the general population have been exposed, it makes interpretation of coronavirus titers difficult. The titers can be elevated due to prior exposure and not from FIP. The titers can only be interpreted in lieu of clinical signs, blood results, etc.

L-lysine is an anti-viral medication that may have some benefit for suppression of herpes virus but would not be a prevention or treatment choice for coronavirus.

The coronavirus is shed in the cat’s feces during active infection with coronavirus. Some infected cats do not shed the virus. The virus attacks the intestinal tract and causes GI upset. PCR on the feces would detect coronavirus, but does not distinguish between the enteric coronavirus and the mutated FIP form of the virus.

How well did you know this?
1
Not at all
2
3
4
5
Perfectly
44
Q

A 10-year old German Shepherd presents to you with the complaint of licking the anal area and scooting. On examination, you find numerous ulcerated tracts in the perianal area that are draining purulent fluid. What is the diagnosis?

  • A) Anal sac impaction
  • B) Anal sac abscess
  • C) Clostridial colitis
  • D) Perianal fistula
A

D) Perianal fistula

This is seen mainly in older German Shepherds, and licking the anus is a common presenting complaint. The key finding is the presence of multiple draining tracts in the perianal region that can actually be quite deep.

How well did you know this?
1
Not at all
2
3
4
5
Perfectly
45
Q

A 2-year-old Quarter horse filly presents with a history of intense pruritis and alopecia in the perineal area (see image). The owners indicated that they noticed the filly rubbing her tail head and perineal area along fences for a period of one week. What diagnostic test will you use to confirm your top differential?

  • A) A fecal flotation to confirm Oxyruis equi
  • B) A scotch tape test to confirm Strongyloides vulgaris
  • C) A fecal flotation to confirm Cyathostomiasis
  • D) A superficial skin scrape to confirm Chorioptes spp.
  • E) A scotch tape test to confirm Oxyuris equi
A

E) A scotch tape test to confirm Oxyuris equi

The correct answer is a scotch tape test to confirm Oxyuris equi, the equine pinworm. While all of these diagnostic tests are important when you suspect parasitism, the scotch tape test will enable you to observe the eggs stuck to the hair. The egg laying activity by the female worm is what causes the intense pruritis. Occasionally, eggs can be found in a flotation, which may give you a false negative interpretation if not found. A superficial skin scrape is a great alternative for this case; however, Chorioptes spp. tend to infest breeds with feathered legs. An infestation of Strongyloides vulgaris or Cyathostomes will likely cause colic, diarrhea, and lethargy.

How well did you know this?
1
Not at all
2
3
4
5
Perfectly
46
Q

Tommy, a 4-year old male neutered domestic short hair, presents to you for frequent urination. Urinalysis reveals 4+ struvite crystals. An abdominal radiograph shows a 1 cm round calculus in the bladder. Tommy does not have a urethral blockage, and urine culture is negative. The owner says surgery to remove this stone is not an option due to finances. Which of the following would be the most important treatment for Tommy?

  • A) Alkalinizing diet
  • B) Potassium citrate
  • C) Metacam
  • D) Clavamox
  • E) Acidifying diet
A

E) Acidifying diet

This cat most likely has a struvite bladder stone. This cannot be determined without a stone analysis, but based on the signalment of the patient, the crystalluria, and the radiograph, this is the most likely type. These types of stones typically form in urine with a high pH. Therefore, an acidifying diet would be appropriate for stone dissolution.

Calcium oxalate stones are the most common type of stone to form in acidic urine. For this type, an alkalinizing diet would be the most appropriate.

Clavamox would only be indicated if this cat had a urinary tract infection sensitive to this antibiotic. This cat had a negative urine culture.

Metacam can be used short-term to help with pain and inflammation associated with the cystitis from this condition but unfortunately Metacam is no longer recommend for use in cats in the United States.

Potassium citrate is a supplement that can be given in addition to an alkalinizing diet to prevent the formation of calcium oxalate stones. This would be contraindicated in this case.

How well did you know this?
1
Not at all
2
3
4
5
Perfectly
47
Q

A 4-year old male Thoroughbred horse presents to you for colic. During your work up, you note a painful enlargement at the root of the mesentery on rectal palpation. You suspect that the cause of the horse’s colic are adults from the egg shown in the picture below. Which of the following drugs effectively kills the adult organisms that can cause this condition?

  • A) Rifampin
  • B) Ivermectin
  • C) Piperonyl butoxide
  • D) Praziquantel
  • E) Metronidazole
A

B) Ivermectin

Colic with an associated painful mass at the root of the mesenery is suspicious for verminous arteritis caused by damage to the cranial mesenteric artery and its branches by Strongylus vulgaris. The strongyle egg shown in the picture confirms the cause in this question. A number of anthelmintics are effective including benzimidazoles, pyrantel and ivermectin.

Praziquantel is effective against tapeworms. Rifampin and metronidazole are antibacterial drugs. Piperonyl butoxide is a pesticide synergist used in insecticide mixtures in horses.

How well did you know this?
1
Not at all
2
3
4
5
Perfectly
48
Q

In canine patients with primary hyperparathyroidism, a chemistry panel would show:

  • A) Hypercalcemia, hyperphosphatemia
  • B) Hypocalcemia, hypophosphatemia
  • C) Hypocalcemia, hyperphosphatemia
  • D) Hypercalcemia, hypophosphatemia
A

C) Hypercalcemia, hypophosphatemia

Patients with primary hyperparathyroidism would have hypercalcemia and normo to hypophosphatemia due to the law of mass action in which phosphorus decreases as calcium increases and vice versa.

How well did you know this?
1
Not at all
2
3
4
5
Perfectly
49
Q

You are examining a 13-year old Standardbred brood mare 5 hours after parturition. You note that the placenta is still present in the reproductive tract of the mare (see image). What is the most appropriate therapy?

  1. Administer penicillin (IM) until the placenta is expelled on its own
  2. Do nothing; the placenta is not considered retained until 12 hours post-parturition and it will likely be expelled by this time
  3. Administer oxytocin (IM or IV) and lavage the uterus to facilitate removal
  4. Place physical traction on the placenta and remove it manually
A
  1. Administer oxytocin (IM or IV) and lavage the uterus to facilitate removal
    * Most texts state that the placenta in a mare is retained after greater than 3 hours post-parturition; therefore, this would be considered a retained placenta. Oxytocin, along with uterine lavage, will cause the uterus to contract and facilitate expulsion of the placenta. Strong physical traction on the retained placenta is generally considered contraindicated, as you may tear the placenta and leave remnants of it within the uterus, resulting in possible complications. Broad-spectrum antimicrobials are often administered to decrease the incidence of metritis but alone would not be appropriate.*
How well did you know this?
1
Not at all
2
3
4
5
Perfectly
50
Q

You are called out to examine a 4-year old dromedary who was castrated 10-days ago and is now not eating. The groin and prepucial area is extremely swollen. The camel is unable to stand and is drooling. His neck is extended and he appears to be unable to swallow. Which of the following is most likely responsible for these symptoms?

  1. Rabies virus
  2. Clostridium tetani
  3. Trypanosoma evansi
  4. Methicillin-resistant Staphylococcus aureus
A
  1. Clostridium tetani

This camel is showing signs of tetanus. The organism was likely introduced into the body through the castration wound. It is recommended that camels are vaccinated against tetanus prior to castration.

While rabies may cause paralysis of the tongue and difficulty swallowing, the recent history of castration in this camel makes tetanus most likely.

Trypanosoma evansi is transmitted via biting flies and causes a slow wasting disease in camels.

How well did you know this?
1
Not at all
2
3
4
5
Perfectly
51
Q

A dog presents with acute onset vomiting, hemorrhagic diarrhea and fever. On fecal examination, you find many large fluke eggs (fr. = oeufs de douve). You question the owner and discover that the dog was recently in Oregon on a boating trip. What agent is most likely causing the clinical signs in this dog?

  1. Nanophyetus salmincola
  2. Neorickettsia helminthoeca
  3. Rickettsia rickettsii
  4. Oxytrema silicula
A
  1. Neorickettsia helminthoeca

The correct answer is Neorickettsia helminthoeca. This rickettsial organism is the causative agent of salmon poisoning. It is carried in the fluke, Nanophyetus salmincola, which requires the snail, Oxytrema silicula in its life cycle. The snail is what confines occurrence of salmon poisoning to the northwest coast.

How well did you know this?
1
Not at all
2
3
4
5
Perfectly
52
Q

A 4-month-old terrier cross presents for inability to eat. The puppy is bright, alert, and responsive on exam. Heart rate is 148, respiratory rate is panting, and temperature is 101.8 F (38.8 C). The puppy is extremely painful when his lower jaw is palpated, and cries and pulls away when you attempt to open his mouth. Sedated oral exam is unremarkable. Radiographs are available for review (see image). What do you tell the owner about prognosis?

  1. This can be cured with antibiotics based on culture and sensitivity. NSAIDs can be used for discomfort.
  2. Chemotherapy can prolong quality of life for a few of months, however prognosis is grave.
  3. Surgical removal of dentigerous cysts will likely be curative, however long term dental disease is common.
  4. This will regress within a year, and symptomatic support is needed for discomfort.
A
  1. This will regress within a year, and symptomatic support is needed for discomfort.
    * Craniomandibular osteopathy (CMO) is seen in young dogs and is exemplified in this radiograph. Terrier breeds predominate, but CMO can be seen in any breed. The disease is self-limiting and regresses typically by 1 year old. It is thought to be a type of hypertrophic osteodystrophy (HOD) and occurs mostly in the mandible but can also affect the tympanic bulla, temporal bones, and temporomandibular joints. Treatment is supportive with nonsteroidal anti-inflammatories to control pain, similar to HOD. Radiographic signs will also regress with time.*
How well did you know this?
1
Not at all
2
3
4
5
Perfectly
53
Q

Closure of the eyes is mediated by cranial nerve ____ and opening the eyes is mediated by CN ____.

  1. III, VII
  2. V, VII
  3. V, III
  4. VII, III
  5. VII, V
A
  1. VII, III

Closure of the eyes is performed by the orbicularis oculi muscle innervated by the facial nerve. Opening of the eye is by the levator palpebrae superioris innervated by the oculomotor nerve.

The trigeminal nerve supplies sensory innervation to the eye.

How well did you know this?
1
Not at all
2
3
4
5
Perfectly
54
Q

A 4 -year old domestic short haired cat presents for anorexia and weight loss of 1 week. Physical exam reveals a body condition score of 7/9, jaundice of the skin and sclera, and dehydration of 4%. Temperature is normal. Bloodwork shows: ALT=303 (25-97 U/L) GGT=1.8 (0-6 U/L) ALP=1170 (0-45 U/L) bilirubin=3.0 (0-0.1 mg/dl) Radiographs show an enlarged liver. What is the most important treatment for the cat’s likely diagnosis?

  1. Parenteral vitamin K injections
  2. Oral clavulanic acid and amoxicillin for 4 weeks
  3. Esophagostomy tube feeding
  4. Oral S-adenosylmethionine for at least 1 month
A
  1. Esophagostomy tube feeding

*The cat described likely has hepatic lipidosis. Cats that are greater than 2 years of age and obese have the greatest risk for hepatic lipidosis. Often these cats are indoor-only and have had a recent stress in their life. An obese cat that is not eating with the above symptoms is most likely to have hepatic lipidosis.

An ALP elevation that is greater in magnitude than GGT is also suggestive of hepatic lipidosis. A bile duct obstruction, cholangiohepatitis, lymphoma, and FIP are other differentials but are less likely with the given information.*

How well did you know this?
1
Not at all
2
3
4
5
Perfectly
55
Q

Which of the following drugs is known for stimulating appetite in small animals?

  1. Omeprazole
  2. Cyproheptadine
  3. Tetracycline
  4. Acyclovir
A
  1. Cyproheptadine

Other appetite stimulants used for stimulating food consumption include mirtazipine, diazepam and oxazepam.

wiki (Cyproheptadine): first-generation antihistamine with additional anticholinergic, antiserotonergic, and local anesthetic properties.

56
Q

A 5-year old male castrated Mastiff presents for left pelvic limb lameness. The medial aspects of both stifles (grassets) are thickened. Manipulation of the left stifle reveals cranial motion of the tibia relative to the femur and a clicking sound from the joint on flexion and extension. What is the most likely diagnosis?

  1. Left caudal cruciate ligament rupture with no meniscal cartilage tear
  2. Left cranial cruciate ligament rupture with no meniscal cartilage tear
  3. Left cranial cruciate ligament rupture with meniscal cartilage tear
  4. Left luxating patella
A

Left cranial cruciate ligament rupture with meniscal cartilage tear

The correct answer is left cranial cruciate ligament rupture with meniscal cartilage tear. The cranial motion of the tibia and medial thickening of the joint (also known as medial buttress) is consistent with the commonly torn cranial cruciate ligament. The clicking heard on flexion and extension is consistent with damaged medial meniscus cartilage found in the stifle.

57
Q

You are doing a summer externship in South America and performing physical exams on a variety of animals. You are performing a fundic exam on the eye of a horse and note what appears to be a worm migrating through the conjunctiva (see image). Because of the location and appearance of this parasite, you suspect this is which of the following?

  1. Toxocara
  2. Thelazia
  3. Dirofilaria
  4. Oxyuris
A
  1. thelazia

Thelazia is a genus of nematode worms (eyeworms) which are found in the ocular tissues. Adults are usually found in the eyelids, tear glands, tear ducts, or the nictitating membrane. They may be found in the eyeball itself under the conjunctiva or in the vitreous. Thelazia are transmitted by Diptera (flies) which do not bite but feed on tears.

Toxocara, which causes ocular larval migrans, usually causes granulomas which may be seen in the retina and appear more circular.

58
Q

An 11-year old Peruvian Paso (cheval) presents with a history of progressive weight loss. Serum chemistry shows elevation in sorbitol dehydrogenase, lactate, alkaline phosphatase, and a decrease in albumin. A vast amount of Crotalaria spp. is seen in the pasture. What type of toxin does Crotalaria spp. possess?

  1. Nitrate
  2. Organophosphate
  3. Pyrrolizidine alkaloid
  4. Cyanide
A
  1. Pyrrolizidine alkaloid

The correct answer is pyrrolizidine alkaloid. The clinical signs described are typical for pyrrolizidine alkaloid toxicity. Consumption of this plant typically results in abnormalities in hepatic cell division; thus resulting in large hepatocytes known as megalocytosis.

59
Q

A 7-month old feedlot steer has died after exhibiting severe fever, dyspnea, cough and respiratory distress. On post mortem there is evidence of fibrinopurulent bronchopneumonia (see image). What bacterium is most likely to be the cause of this syndrome?

  1. Mannheimia hemolytica
  2. Trueperella pyogenes ( formerly Arcanobacterium pyogenes)
  3. Bovine herpes virus type 4
  4. Mycoplasma bovis
  5. Pasteurella multocida
A
  1. Mannheimia hemolytica

Other agents may also be isolated, but this is recognized as the worst pathogen in bovine pulmonary disease. It was formerly called Pasteurella hemolytica.

60
Q

A 1.5-year-old spayed female cat presents with a 7-day history of vocalizing, rolling, and allowing a male neutered cat in the household to mount her. The cat has gone through one similar episode 1 month ago. She otherwise has been behaving normally and has no other health problems. You performed an ovariohysterectomy on the cat at 3 months of age. You perform vaginal cytology which shows some cornified epithelial cells but is inconclusive. You measure serum lutenizing hormone of 0.2 ng/ml (normal for an ovariectomized female is >1 ng/ml). Which of the following is the most appropriate course of action?

  1. Exploratory laparotomy to remove the ovarian remnant
  2. Order MRI of the brain to rule out a pituitary tumor
  3. Measure serum estrogen and progesterone levels
  4. Measure serum testosterone levels
A
  1. Exploratory laparotomy to remove the ovarian remnant
    * This is a classic description of ovarian remnant syndrome, which is when a cat goes into estrus after previously having an ovariohysterectomy (OVH).

This can occur anywhere from weeks to years after OVH and typically the clinical signs consistent with estrus are sufficient to conclude that the cat is in estrus and has ovarian tissue present. Additional diagnostic tests that are consistent with ovarian remnant syndrome include:

Serum estrogen >70 pmol/L indicate that the cat has estrogen production from the ovary. The problem with this test is that estrogen measurements may fluctuate and can be unreliable.

Serum progesterone >6 nmol/L after induced ovulation is sufficient to conclude that corpora lutea formed and released progesterone.

Testing for serum LH levels can also help confirm the diagnosis. In intact queens, LH is consistently maintained at basal levels due to negative feedback from ovarian estradiol secretion. After OVH, this control is lost and LH concentrations increase. LH <1 ng/mL is consistent with the presence of an ovary as it is in this case.

Surgery is the treatment of choice. Many practitioners prefer to do surgery during estrus or diestrus when the ovarian tissue is enlarged and easier to locate. Remnants may be bilateral so a complete exploratory laparatomy is necessary. Ovarian tissue is most commonly at the ovarian pedicle but can also be in the mesentery or elsewhere.*

61
Q

A 2-year old male castrated mixed breed dog presents for an altered gait after being hit by a car. On physical exam, the thoracic limbs had decreased biceps and triceps reflexes and decreased muscle tone. The pelvic limbs had hyper-reflexive patellar and gastrocnemius reflexes and increased muscle tone. Where is the spinal cord lesion?

  1. T3-L3
  2. C6-T2
  3. C1-C5
  4. L4-S3
A
  1. C6-T2
  • The correct answer is C6-T2. In a dog, the spinal cord is divided into the four regions listed above. Upper motor neuron signs include hyper-reflexia of spinal reflexes and increased muscle tone. Lower motor neuron signs include decreased or absent spinal reflexes and decreased muscle tone. A lesion between the C6-T2 spinal cord segments would result in lower motor neuron signs in the thoracic limbs and upper motor neuron signs in the pelvic limbs, as described in the patient in the question.*
  • A lesion between C1-C5 spinal cord segments would manifest as upper motor neuron signs in the thoracic and pelvic limbs. Lesions between T3-L3 spinal cord segments would not affect the thoracic limbs, but would manifest as upper motor neuron signs in the pelvic limbs. Lesions between L4-S3 spinal cord segments would result in normal thoracic limbs and lower motor neuron signs in the pelvic limbs.*
62
Q

Which of the following is a causative agent of infectious bovine keratoconjunctivitis as shown in this image?

  1. Histophilus somni
  2. Moraxella bovis
  3. E. coli
  4. Thelazia
A
  1. Moraxella bovis

The correct answer is Moraxella bovis. Thelazia is the eye worm. Histophilus somni is a cause of many syndromes, but is rarely found in the eye. E. coli is not an ocular pathogen.

63
Q

A horse presents to your clinic after ingesting a large amount of grain. What is your major concern?

  1. Laminitis
  2. Impaction
  3. Torsion
  4. Choke
  5. Acidosis
A
  1. laminitis

The correct answer is laminitis secondary to endotoxemia. Laminitis, endotoxemia, and diarrhea are commonly associated with grain overload and appropriate therapy to evacuate any remaining stomach contents, ameliorate endotoxin and prevent laminitis should be instituted immediately. Laxatives such as mineral oil are commonly administered.

64
Q

In canine patients with primary hyperparathyroidism, a chemistry panel would show:

  1. Hypercalcemia, hyperphosphatemia
  2. Hypocalcemia, hyperphosphatemia
  3. Hypocalcemia, hypophosphatemia
  4. Hypercalcemia, hypophosphatemia
A
  1. Hypercalcemia, hypophosphatemia

The correct answer is hypercalcemia, hypophosphatemia. Patients with primary hyperparathyroidism would have hypercalcemia and normo to hypophosphatemia due to the law of mass action in which phosphorus decreases as calcium increases and vice versa.

65
Q

Tommy, a 4-year old male neutered domestic short hair, presents to you for frequent urination. Urinalysis reveals 4+ struvite crystals. An abdominal radiograph shows a 1 cm round calculus in the bladder. Tommy does not have a urethral blockage, and urine culture is negative. The owner says surgery to remove this stone is not an option due to finances. Which of the following would be the most important treatment for Tommy?

  1. Metacam
  2. Clavamox
  3. Potassium citrate
  4. Acidifying diet
  5. Alkalinizing diet
A
  1. Acidifying diet
    * This cat most likely has a struvite bladder stone. This cannot be determined without a stone analysis, but based on the signalment of the patient, the crystalluria, and the radiograph, this is the most likely type. These types of stones typically form in urine with a high pH. Therefore, an acidifying diet would be appropriate for stone dissolution.

Calcium oxalate stones are the most common type of stone to form in acidic urine. For this type, an alkalinizing diet would be the most appropriate.

Clavamox would only be indicated if this cat had a urinary tract infection sensitive to this antibiotic. This cat had a negative urine culture.

Metacam can be used short-term to help with pain and inflammation associated with the cystitis from this condition but unfortunately Metacam is no longer recommend for use in cats in the United States.

Potassium citrate is a supplement that can be given in addition to an alkalinizing diet to prevent the formation of calcium oxalate stones. This would be contraindicated in this case*
.

66
Q

What is the most common cause of maxillary sinusitis in a horse, as seen in the necropsy image below?

  1. Brachygnathia
  2. Dentigerous cyst
  3. Foreign body
  4. Tooth root abscess
  5. Guttural pouch mycosis
A
  1. Tooth root abscess

correct answer is tooth root abscess. Commonly the first molar teeth are involved. Clinical signs include weight loss, quidding (dropping half chewed feed), halitosis, swelling, and unilateral purulent nasal discharge.

67
Q

The image shows an infarct in the liver discovered on post mortem exam of a mature beef cow which died one hour before in a western mountain pasture, after being observed to appear normal one day earlier. There is also dark red urine in the bladder. The pasture contains native plants, some pine trees, and a marshy area with water plants. The cows are unvaccinated and were never wormed. Given this history and the lesion found, the most likely cause of death is _________.

  1. Viral hepatitis
  2. Death camas toxicity
  3. Pine needle poisoning
  4. Blue-green algae toxicity
  5. Bacillary hemoglobinuria
A
  1. Bacillary hemoglobinuria

Also known as redwater, bacillary hemoglobinuria is caused by germination of Clostridium Novyi type D spores in the liver after anaerobic damage by migrating liver fluke larvae (douve). Cl. Novyi was formerly called Cl. hemolyticum. Vaccination can prevent this disease.

68
Q

A 4-year old male castrated Basenji presents for polyuria, polydipsia, and weight loss. Blood work shows P=2.5 (2.9-5.3 mg/dl), K= 3.1 (3.9-5.1 mEq/L), total CO2= 12 (17-25 mmol/L). The remainder of the blood work is within normal limits. Urinalysis shows 3+ glucose. Which of the following is your most likely differential diagnosis?

  1. Pyelonephritis
  2. Pyometra
  3. Fanconi syndrome
  4. Diabetes mellitus
A
  1. Fanconi syndrome

The correct answer is Fanconi syndrome. Fanconi syndrome is an inherited disease in Basenjis. The disease involves renal tubular defects causing an abnormal loss of electrolytes and solutes leading to hypophosphatemia, hypokalemia, and metabolic acidosis. DM is less likely because serum glucose is normal. The lab abnormalities present in this dog are not consistent with pyelonephritis. Pyometra is not a viable choice as the signalment describes a male.

69
Q

A chicken operation has recently been ravaged by a respiratory disease affecting almost all of the chickens in the flock. The chickens are coughing and sneezing and many have facial swelling. You necropsied many of the chickens and found mucoid exudate in the bronchi, thickened air sacs, and in a few of the chickens, interstitial nephritis was present. Which of these diseases is likely?

  1. Infectious bronchitis
  2. Fowl cholera
  3. Infectious bursal disease
  4. Aspergillus
A
  1. Infectious bronchitis

The correct answer is infectious bronchitis. This is caused by a coronavirus. It is spread by aerosol and ingestion and usually affects all exposed birds. The clinical signs and necropsy findings are as described in the question. The disease can be clinically indistinguishable from mild forms of Newcastle disease, laryngotracheitis, and infectious coryza. Virus isolation is needed to obtain a definitive diagnosis.

70
Q

Which of these drugs should not be used in Greyhounds?

  1. Ivermectin
  2. Morphine
  3. Propofol
  4. Thiopental
A
  1. Thiopental

The correct answer is thiopental. Thiopental is an ultra-short acting barbiturate. Recovery depends on redistribution to tissues, including fat. Because sighthounds have very little fat, they have prolonged recoveries and greater complications with these drugs.

71
Q

Severe combined immunodeficiency is a lethal autosomal recessive trait in Arabian foals. Heterozygotes are clinically normal. If the heterozygote carrier rate for the genetic mutation is 8%, what is the expected frequency of Arabian foals that are homozygous for the mutated allele?

  1. 4%
  2. 25%
  3. 2%
  4. 0.064%
  5. 0.64%
  6. 0.16%
A
  1. 0.16%

You do not need to know anything about the disease in this question in order to get the correct answer. You are told that the disease is recessive and has a carrier rate of 8%

Because the trait is recessive, homozygotes will be the only individuals affected. In order for a foal to be born homozygous for the trait, BOTH parents MUST be carriers. The chances of both parents being carriers is 8% x 8% (or 0.08 x 0.08) = 0.0064 or 0.64%. If both parents are carriers, the offspring has a 1 in 4 chance of inheriting two mutant alleles (50% chance for each allele from each parent).

Since the chances of both parents being carriers is 0.64% and the chance of having a homozygous offspring in that case is 1 in 4, the overall expected frequency of diseased foals is 0.0064 x 0.25= 0.0016 or 0.16%.

72
Q

A 16-year old budgerigar presents with progressive unilateral paresis of the right leg (see image). There is normal flexion and extension of the hip joint with decreased flexion, extension, and sensation below the knee. The remainder of your skeletal and neurologic examination is unremarkable. Which of the following is most likely in this bird?

  1. Lead toxicosis
  2. Botulism
  3. Marek’s disease
  4. Intervertebral disc disease
  5. Renal neoplasia
A
  1. Renal neoplasia

The signalment and progressive unilateral paresis are most suspicious for renal neoplasia of budgerigars. Affected birds develop paresis due to pressure exerted on the sciatic nerve by a renal tumor. There is typically normal flexion and extension of the hip joint with decreased flexion, extension, and sensation below the knee as described here.

Disc disease is uncommon in birds. Lead toxicity can lead to peripheral neuropathy and paresis but typically there are more multifocal or other accompanying signs such as polyuria/polydipsia, gastrointestinal signs, wing droop, head tilt, or convulsions. Marek’s disease typically affects younger animals. Botulism typically presents with flaccid paralysis of the legs, wings, and neck.

73
Q

You need to perform a CBC and take thoracic radiographs on a somewhat fractious cat. You sedate the cat with an intramuscular injection of medetomidine. While on the X-ray table, you become concerned that the cat is not doing well and you decide you want to reverse the effects of medetomidine. What should you give the cat?

  1. Xylazine
  2. 2-pralidoxime
  3. Flumazenil
  4. Atropine
  5. Atipamezole
A
  1. atipamezole

Atipamezole (trade name: Antisedan), an alpha-2 antagonist, is the reversal agent for medetomidine (trade name: Dormitor). Medetomidine is an alpha-2 agonist.

Xylazine would be another example of an alpha-2 agonist. Yohimbine is its reversal agent. Other alpha-2 agonists are clonidine, detomidine, dexmedetomidine and romifidine

2-pralidoxime is a reversal agent for cholinesterase inhibitors.

Flumazenil is a reversal agent for benzodiazepines

Atropine is a mAChR antagonist and may be dangerous to use after administration of alpha-2 agonists. This is because alpha-2 agonists cause marked vasoconstriction and high afterload on the heart. Giving atropine and increasing the heart rate can place further stress on the heart. You do not want to set in motion peripheral vasoconstriction and compensatory bradycardia brought on by the alpha-2, then increase the heart rate against that high afterload. This could make for an unhappy heart in the event underlying subclinical heart disease is present.

74
Q

What profile would you expect in a dog with hypervitaminosis D?

  1. Low Ca, High P
  2. Low Ca, Low P
  3. High Ca, High P
  4. High Ca, Low P
A
  1. High Ca, High P

Excessive intake of vitamin D is associated with an increase in 25-hydroxyvitamin D3 levels. At high levels, 25-hydroxyvitamin D3 competes with 1,25-dihydroxyvitamin D3 for its receptors on the intestines and bone causing increased absorption of Ca and P from the intestinal tract and resorption of bone causing increased levels of circulating Ca and P. A common source of confusion is that this is in contrast to PTH which causes high Ca but generally causes unchanged or normal phosphorus because it also enhances renal phosphorus excretion

75
Q

Which of these would be appropriate for a horse with hyperkalemic periodic paralysis (HYPP)?

  1. Brome hay
  2. Timothy hay
  3. Alfalfa hay
  4. Beet molasses
A
  1. Timothy hay

Of these choices, the only feed with low potassium is timothy hay. A low potassium diet is the most important nutritional modification in the treatment of HYPP. Regular exercise and feeding smaller, frequent meals can also reduce clinical signs. This disease is inherited in an autosomal dominant fashion, and owners should be discouraged from breeding affected animals.

76
Q

Black walnut toxicity causes what condition in horses?

  1. Pulmonary edema
  2. Colic
  3. Hypersalivation
  4. Laminitis
A
  1. Laminitis

The correct answer is laminitis. Although the specific toxic principle in black walnuts responsible for causing laminitis or acute lameness in horses is unknown, evidence suggests juglone (a napthaquinone) plays a role. Do not use black walnuts for horse bedding.

77
Q

A 2-year old male castrated Border Collie presents for a 1-week history of small bowel diarrhea. A fecal flotation shows numerous Giardia cysts. What is the treatment of choice for this dog?

  1. Decoquinate
  2. Ipronidazole
  3. Albendazole
  4. Metronidazole
A
  1. Metronidazole

The correct answer is metronidazole. You should know that fenbendazole is actually the treatment of choice for Giardia, but metronidazole has historically been the most widely accepted option. If fenbendazole is not offered as an answer choice, choose metronidazole. If fenbendazole were offered as an answer choice, it would be the best option.

78
Q

You perform an abdominal ultrasound to evaluate a Yorkshire Terrier with a suspected liver shunt. After finding the shunt vessel, you complete the ultrasound and find several calculi in the bladder. What is the most likely type of stone?

  1. Xanthine
  2. Cysteine
  3. Calcium oxalate
  4. Urate
  5. Struvite
A
  1. Urate

The correct answer is urate. Animals with portosystemic shunts are very predisposed to developing urate uroliths due to their inability to metabolize purines appropriately.

79
Q

A 7-year old female spayed Standard Poodle presents with weakness and lethargy. A chemistry panel shows a Na+ = 130 mEq/L (142-152 mEq/L), K+ = 6.5 mEq/L (3.9-5.1 mEq/L), BUN 55 mg/dl (8-28 mg/dl), creatinine 1.9 mg/dl (0.5-1.7 mg/dl). The test to run for a definitive diagnosis would be which of the following?

  1. Thoracic radiographs
  2. Bile acids test
  3. ACTH stimulation test
  4. Electrocardiogram
A
  1. ACTH stimulation test

The correct answer is ACTH stimulation test. The chemistry profile above is highly suggestive of hypoadrenocorticism. An ACTH stimulation test would give a definitive diagnosis for Addison’s disease if the plasma cortisol concentration is low after ACTH administration.

A bile acids test is not indicated since liver function is not in question. Thoracic radiographs may show microcardia, hypoperfused lungs, and rarely megaesophagus. ECG abnormalities would be consistent with hyperkalemia, which includes wide, flat, or absent P waves, widened QRS complexes, tall spiking T waves, and bradycardia.

(BUN et créatinine sont élevés car perte de Na = perte d’eau = déshydratation pré-rénale (hypoperfusion)

80
Q

A 4-year old female spayed Siberian Husky presents for further evaluation as a result of developing crusting and hyperkeratosis around the eyes, nose, and mouth. A skin scraping did not identify an etiology. How would you treat this dog?

  1. Ivermectin treatment
  2. Corticosteroids
  3. Zinc supplementation
  4. Daily wiping with chlorhexidine pads
A
  1. Zinc supplementation

Zinc responsive dermatopathy occurs most frequently in the Siberian Husky. Other species such as cattle, goats, and humans have a similar condition. The exact cause is unknown, but it has been linked to defective intestinal absorption of zinc. Lesions may vary in severity. The response is typically seen within days. The key to answering this question appropriately was to recognize the signalment of the patient, the location of the lesions which is characteristic of zinc responsive dermatopathy, and the fact that there was no infectious etiology such as demodex identified.

81
Q

A 2-year old Quarter Horse presents for intermittent muscle fasciculation followed by weakness. What condition should you suspect?

  1. Stringhalt
  2. Myotonia
  3. Hyperkalemic periodic paralysis
  4. Tetanus
  5. Grass tetany
A
  1. HYPP

The correct answer is hyperkalemic periodic paralysis (HYPP). HYPP is seen in Quarter Horses due to a point mutation in a key part of a skeletal muscle sodium channel subunit (wiki: The mutation causes single amino acid changes in parts of the channel which are important for inactivation. These mutations impair “ball and chain” fast inactivation of SCN4A (gene codes for a voltage-gated sodium channel Nav1.4) following an action potential). This results in elevation of the resting membrane potential to increase the likelihood of depolarizing. Excess concentrations of potassium can result in failure of the sodium channels to inactivate (K+ doit sortir des cellules pour que la cellule retourne à son RMP de -70mv, difficile si K+ extracellulaire est trop élevé). Therefore, treatment is directed at decreasing dietary potassium.

82
Q

A 4-year old domestic short haired male cat presents for an acute onset of vomiting and anorexia of one day duration. On physical exam you note the cat is depressed and approximately 5% dehydrated. Radiographs show an abnormally increased amount of plication of the small intestines. What is your most likely diagnosis?

  1. Hepatic lipidosis
  2. Panleukopenia
  3. Pancreatitis
  4. Lymphoma
  5. String foreign body
A
  1. String foreign body

The correct answer is string foreign body. The radiographic description is classic for string foreign bodies. Panleukopenia is not likely because of the radiographic findings, and you would also expect more systemic signs such as respiratory disease and fever. With pancreatitis cats do not vomit as commonly as dogs. They usually show vague nonspecific signs. Hepatic lipidosis would not result in an acute onset of vomiting

83
Q

A budgerigar is showing signs of squamous metaplasia of the oral mucosa, conjunctiva, and upper airways. It has developed associated bacterial sinusitis (see image). In a pet bird, what are these clinical signs most suggestive of?

  1. Hypervitaminosis D
  2. Hypovitaminosis D
  3. Iodine deficiency
  4. Hypocalcemia
  5. Hypovitaminosis A
A
  1. Hypovitaminosis A

The correct answer is hypovitaminosis A. The functions of vitamin A are related to epithelial maintenance, vision, and skeletal development. In birds, problems with the skeleton or vision are rarely seen with vitamin A deficiency. Vitamin A deficiency can manifest as squamous metaplasia of the oral mucous membranes or glands. Glands may be entirely converted to squamous epithelium with keratin material and can look like abscesses or pustules but are essentially keratin cysts. They should be differentiated from lesions of pox, Candida, and Trichomonas. Lesions of the conjunctiva, nasolacrimal duct, upper GI tract, and upper respiratory tract can occur. Presenting signs may include severe dyspnea or respiratory signs

84
Q

Which of the following zoonotic pathogens is most likely to be acquired via raw goat milk?

  1. Brucella suis
  2. Brucella melitensis
  3. Bacillus anthracis
  4. Clostridium difficile
  5. Brucella abortus
A
  1. B. melitensis

B. melitensis is a severe pathogen in humans, as are B. abortus from cattle, B. suis from pigs and B. canis from dogs.

85
Q

In January you examine a group of dairy calves which range in age from 2 to 7 months, with a complaint of hair loss and pruritus. The calves are thin and mucous membranes are pale. One has developed bronchopneumonia, and is also febrile and depressed. You do a skin scraping and find the parasite shown in the image, which your technician identifies as Solenopotes sp. The CBC shows the calves to be severely anemic. What treatment recommendation should you now make to the dairy owner?

  1. Vaccinate all calves against Mannheimia hemolytica
  2. Treat all calves for lice
  3. Treat all calves with long acting tetracycline
  4. Treat all calves for mange
  5. Treat all calves with hematinics
A
  1. Treat all calves for lice

This is a blood sucking genus of louse, and can cause severe anemia. The anemic calves become thin and more susceptible to diseases like pneumonia. You can tell this is a louse and not a tick or a mite because lice are insects with 6 legs and ticks and mites are arachnids with 8 legs.
The three genera of blood sucking cattle lice are Solenopotes, Linognathus, and Hematopinus.

86
Q

A 10-year old German Shepherd presents to you with the complaint of licking the anal area and scooting. On examination, you find numerous ulcerated tracts in the perianal area that are draining purulent fluid. What is the diagnosis?

  1. Perianal fistula
  2. Anal sac abscess
  3. Anal sac impaction
  4. Clostridial colitis
A
  1. Perianal fistula

The correct answer is perianal fistula. This is seen mainly in older German Shepherds, and licking the anus is a common presenting complaint. The key finding is the presence of multiple draining tracts in the perianal region that can actually be quite deep.

87
Q

Suzie-Q, a 6-month old female spayed domestic short hair was recently adopted from the humane society. She has had watery diarrhea since adoption. Her fecal float and Giardia ELISA tests were negative. She was treated with metronidazole with no clinical improvement. You soak a cotton tip swab with saline and swab the rectum. You see elongated motile oval shaped protozoan organisms that do not look like Giardia lamblia. What organism might this be and what is the appropriate therapy?

  1. Tritrichomonas foetus, Ronidazole
  2. Paragonimus kellicotti, Praziquantel
  3. Cryptosporidium, Clindamycin
  4. Enterobius vermicularis, Fenbendazole
  5. Taenia taeniaformis, Praziquantel
  6. Giardia intestinalis, Fenbendazole
A
  1. Tritrichomonas foetus, Ronidazole

*Tritrichomonas foetus is a flagellated parasite most commonly found in kittens that have had an unresponsive diarrhea. The parasite can be very difficult to diagnose. It is most often responsive to Ronidazole.

Paragonimus is a lung fluke. The eggs are typically passed in the feces. Fenbendazole and Praziquantel have been effective against this parasite.

Giardia is unlikely if the ELISA is negative as it is a very sensitive test.

Taenia is a tapeworm and is not a flagellated parasite. It is treated with Praziquantel.

Cryptosporidium is a coccidian that invades the small intestinal villi after ingestion of infected oocysts. It can be diagnosed with PCR. It is treated with clindamycin, azithromycin, or tylosin most commonly. It is usually an opportunist, so evaluation for underlying disease is appropriate.

Enterobius vermicularis, or pinworm, is a parasite of people and primates but not cats and dogs.*

88
Q

A 1.5-year old Quarter Horse gelding is presented to you for symmetric ataxia, weakness, and spasticity of all limbs, but worse in the hind limbs. When walking, the horse frequently drags his toes and the hind limbs frequently interfere with one another. Based on the signalment, history and physical examination findings, which of the following is the most likely cause of these clinical sign

  1. Botulism
  2. Equine Motor Neuron Disease (EMND)
  3. Equine Protozoal Myeloencephalitis (EPM)
  4. Equine Degenerative Myeloencephalopathy (EDM)
  5. Cauda Equina Syndrome
A
  1. EDM

*The horse in this question has clinical signs most consistent with EDM; cervical vertebral malformation (wobblers) is also a possibility, but was not provided as an answer. The cause of EDM is unknown, but this disease typically affects young horses (< 2-3 years of age; but older horses can develop disease). Clinical signs are a result of diffuse neuronal fiber degeneration of various portions of the central nervous system. This disease has been associated with low serum vitamin E concentrations, suggesting that oxidative damage may play a role in the development of disease.

EMND is typically associated with muscle tremors, shifting of weight while standing, muscle atrophy and recumbency. Botulism is associated with generalized muscle weakness. Cauda equina syndrome causes analgesia of the perineum. EPM can cause a range of clinical signs, but is typically with asymmetric neurologic deficits.*

89
Q

A client brings the one-half inch grub shown in the image to you one spring day, telling you it emerged from a hole in the back of one of his prize show cattle. He wants to know what to treat his cattle with and when.

  1. Moxidectin now, in spring
  2. Organophosphates in summer
  3. Thiabendazole in early fall
  4. Ivermectin in February
  5. Ivermectin in early fall
A
  1. Moxidectin now, in spring

This is a cattle warble called Hypoderma. The 2 species are H. bovis and H. lineatum. They undergo a long migration in tissues and only emerge from the back of the animal in spring. The crucial treatment time is early fall when l_arvae are just beginning to migrate_ in tissues. Organophosphates or one of the macrocyclic lactones (ivermectin, doramectin, eprinomectin or moxidectin) are effective.

90
Q

A 6-year old West Highland White Terrier comes in to see you for the mucopurulent ocular discharge as seen in the photo below. A Schirmer tear test shows no tear production. What is the treatment of choice for chronic canine keratoconjunctivitis sicca?

  1. Topical cyclosporine and a topical steroid
  2. Systemic antibiotics and corticosteroids
  3. Topical cyclosporine and systemic corticosteroids
  4. Systemic cyclosporine and antibiotics
A
  1. Topical cyclosporine and a topical steroid

The correct answer is topical cyclosporine and a topical steroid. The treatment of KCS is aimed at reducing immune destruction of the lacrimal glands. Topical cyclosporine (Optimmune) and a topical steroid (frequently in a triple antibiotic/steroid ointment) are the treatment of choice. You should be cautious using steroids in acute cases due to the risk of corneal ulceration.

91
Q

You are asked to perform a necropsy on a 17-year old Standardbred mare on a large horse ranch. Although not related to the cause of death, you notice the parasite shown in the image within the stomach. You tell the owner that this parasite is also responsible for the eggs that he sees seasonally on the hair of the front legs of his horses. The owner asks what should be done about this. You discuss the importance of promptly cleaning up feces and transporting feces away. In addition, which of the following is the most appropriate recommendation for ongoing control of this parasite?

  1. Administer fenbendazole twice annually, once in the early summer and again in late summer
  2. Administer fenbendazole twice annually, once in the early spring and again in the fall
  3. Administer ivermectin twice annually, once in the early spring and again in the winter
  4. There is no need to treat these parasites because they are not associated with disease in horses
  5. Administer ivermectin twice annually, once in the early summer and again in the fall
A
  1. Administer ivermectin twice annually, once in the early summer and again in the fall

*This case describes the appearance of the horse bot fly, Gasterophilus spp. Gasterophilus is frequently asymptomatic but treatment is recommended because bots can cause gastritis and frequently are a source of annoyance and stress to horses. In addition, the larval instars can cause stomatitis, and colic.

The key to answering this question regarding optimal treatment and management is an understanding of the Gasterophilus life cycle. Gasterophilus undergoes complete metamorphosis, including three larval instars and only one generation is produced per year. The general cycle typically begins with the female ovipositing 150-1,000 eggs on a horse during the early summer months directly on single hairs of the horse’s front legs (especially around the cannon bone area) as well as the abdomen, flanks, and shoulders. The eggs are approximately 1-2mm long and are pale to grayish yellow attached near the tip of the hair.

The eggs develop into first instar larvae within five days and they are stimulated to emerge by the horse li_cking or biting at the fully developed eggs. The larvae then crawl to the mouth or are ingested and subsequently bury themselves in the tongue or gingiva and remain for approximately 28 days. The larvae molt to the second stage and move into the stomach. The second and later third stage larvae typically attach to the lining of the stomach in the non-glandular portion near the junction of the esophageal and cardiac regions where they r_emain immobile for the following 9 to 12 months.

The third instar larvae are relatively large, between 1-2cm long with a rounded body, narrow, hooked mouthparts, and spines. The hooked mouthparts enable the larvae to securely attach to the lining of the stomach and intestinal tract. After the third instar larvae have matured, they detach from the gastrointestinal tract and pass from the horse’s body in the f_eces._ The larvae burrow into the soil or dried manure where they pupate and remain for the next one to two months. This stage of the life cycle occurs between late winter and early spring.

Based on this life cycle, the recommended management protocol is typically to treat with an avermectin to control adults and all larval stages by administering in the early summer, shortly after any eggs are seen and again in the fall at the end of the botfly season. Such a control program will substantially reduce fly numbers.*

92
Q

A middle aged MN stray cat is left on the doorstep of your clinic. The cat has a large dry crusted area of alopecia over his nose. A skin scraping of the area is negative. The lesion fluoresces under Wood’s lamp examination (see image). Which of the following would be the best treatment?

  1. Doxycycline
  2. Lufenuron
  3. Full body lyme sulfur dip, itraconazole
  4. Athlete’s foot cream (clotrimazole)
  5. Povidone-iodine scrub
A
  1. Full body lyme sulfur dip, itraconazole

*This cat has a ringworm infection caused by Microsporum canis. This fungi fluoresces blue under a Wood’s lamp in 50% of cases.

The best treatment for ringworm infection would include a combination topical and oral therapy. Lyme sulfur dip or an antifungal shampoo containing miconazole would be acceptable. Oral antifungals such as itraconazole or fluconazole are most effective with the least side effects.

Povidone-iodine scrub has not been shown to be effective against ringworm.

Lufenuron is classified as an insect development inhibitor because of its ability to inhibit chitin synthesis, thus in the past has been said to have some effect against fungal infections. This has been debated and not widely supported as a treatment for ringworm.

Doxycycline is an antibiotic and would not be effective in treatment of fungal disease.

Athlete’s foot cream (clotrimazole) may have some effect at treating the lesion. Most over-the-counter creams such as this also include a steroid like betamethasone which would not be desired. Although this lesion appears to be localized to the nasal area, ringworm may also be subclinical and this cat may have infection elsewhere in the skin that is not grossly visible. Therefore, the best therapy is a combination of topical and oral.*

93
Q

Which of these drugs has the greatest potential for causing acute renal failure in the horse?

  1. Oxytetracycline
  2. Dexamethasone
  3. Diphenhydramine
  4. Xylazine
  5. Neomycin
A
  1. Neomycin

The correct answer is neomycin. Aminoglycosides are one of the most common causes of renal tubular nephrosis and acute renal failure. Of the aminoglycosides, neomycin is probably the most nephrotoxic, followed by gentamicin, amikacin, and streptomycin. The other big class of nephrotoxic drugs is non-steroidal anti-inflammatory drugs.

94
Q

A 7-year old male intact Chesapeake Bay Retriever presents to your clinic with the presenting complaint of an intermittent cough. On exam, the dog is bright and alert with a temperature of 100.5F (38.1 C), heart rate of 110 beats per minute and respiratory rate of 30 breaths per minute. You perform chest radiographs which are shown below. A CBC shows a hematocrit of 39% (35-57%), neutrophil count of 8,659/ul (2,900-12,000/ul), monocyte count of 984/ul (100-1,400/ul) and eosinophil count of 1,980/ul (0-1,300/ul). What is the treatment of choice for the most likely diagnosis?

  1. Furosemide
  2. Terbutaline
  3. Pericardiocentesis
  4. Immiticide (Melarsomine)
  5. Enrofloxacin
A
  1. Immiticide (melarsomine)

*The correct answer is Immiticide (melarsomine). The dog in this radiograph has the classic findings for heartworm disease, including right sided ventricular enlargement with very prominent pulmonary arteries.

Heartworm in the dog is caused by Dirofilaria immitis. The treatment of choice for heartworm is Immiticide which is given by intramuscular injection.

Furosemide is a treatment for congestive heart failure, which this dog shows no evidence of. Terbutaline is a bronchodilator, which would be of little use in this case. Broad-spectrum antibiotics are useful for the treatment of pneumonia and other infections but are not effective against heartworm. Pericardiocentesis is obviously the treatment for pericardial effusion, but this dog’s radiographs do not show the classic globoid heart you might see with pericardial effusion.

Adulticidal treatments for heartworm are melarsomine and thiacetarsamide.*

95
Q

An owner brings her 4-year old female Labrador Retriever to your clinic because she believes she might be pregnant. She does not remember when the dog’s last heat cycle was. The dog’s abdomen appears fairly distended and you take a lateral abdominal radiograph which is shown below. What would be the earliest time you would expect to be able to see fetal skeletons on abdominal radiographs in the dog?

  1. 43 days gestation
  2. 33 days gestation
  3. 23 days gestation
  4. 53 days gestation
A

The correct answer is 43 days gestation. The fetal skeleton ossifies at 42-45 days in the dog and 35-39 in the cat.

In the dog, a mineralized fetus can usually be seen around 42-46 days. The scapula, humerus, and femur can be made out around 46-51 days. The ribs can be seen at 52-59 days. Teeth and toes can be seen at 58-63 days.

96
Q

A 6-year old Friesian cow presents to you with a mass on the left mandible (see image). The farmer reports that the mass has developed over the last several weeks and the cow has recently had some difficulty eating and lost weight. On examination, the mass is firm, immobile, and painful on manipulation. You note a thick discharge with small granular particles. Based on the most likely diagnosis, what should you tell the farmer?

  1. Treatment is unlikely to be successful and the cow should be culled
  2. Treatment with sodium iodide intravenously is likely curative but the cow’s milk and meat will not be suitable for human consumption for 120 days
  3. The most effective treatment is oral sodium iodide
  4. Intravenous penicillins are likely to be effective
A
  1. Treatment is unlikely to be successful and the cow should be culled

This is a case of Actinomyces bovis or “lumpy jaw”. The keys to the diagnosis are the firm mass that is immobile over the mandible. The presence of “sulfur granules” in the discharge is also a characteristic finding with this disease. Unfortunately, simply making the diagnosis is not sufficient to answer this question correctly. All of the answer choices are potential treatments for lumpy jaw but this represents a moderate to severe case which is unlikely to respond to treatment due to difficulty in achieving the necessary antibiotic concentration over a sustained period. Therefore, the best answer choice in this case is to cull the cow.

97
Q

What is the most common neoplasia seen in the equine stomach?

  1. Gastric adenocarcinoma
  2. Mesothelioma
  3. Squamous cell carcinoma
  4. Lymphosarcoma
A
  1. Squamous cell carcinoma (SCC, carcinome des cellules basales)
98
Q

The cat in the image below presents for lethargy, depression, and weakness. The cat can’t seem to lift his head (as seen in this image). You recommend running a chemistry panel on the cat. What potential finding explains can explain the findings?

  1. Low potassium
  2. Low calcium
  3. Low phosphorus
  4. High urea nitrogen
  5. High glucose
A
  1. Low potassium

*The cat in the image is exhibiting cervical ventroflexion, which is a general sign of weakness. The most common cause for this presentation is hypokalemia (low potassium), which can be caused by a variety of reasons (such as chronic renal failure).

This weakness can also be caused by a number of different problems other than low potassium, including myasthenia gravis; polymyopathies caused by toxoplasmosis, immune-mediated disease, or hyperadrenocorticism; and neuropathies caused by organophosphate poisoning, thiamine deficiency, or botulism.*

99
Q

Several litters of 2 to 3 day old pigs have recently died rapidly with hemorrhagic enteritis. Post mortem lesions include mucosal hemorrhage, necrosis and emphysema in the small intestines. What pathogen is most likely to cause these signs?

  1. Enterotoxigenic E. coli
  2. Haemophilus parasuis
  3. Erysipelothrix rhusiopathiae
  4. Clostridium perfringens type C
  5. Salmonella choleraesuis
A
  1. Clostridium perfringens type C

The age of these affected pigs along with the rapid course of hemorrhagic and necrotic enteritis help you come to this conclusion. As with many Clostridial diseases, vaccination is the most effective means of control. Other important clostridial diseases of swine include C. perfringens type A, C. difficile, C. tetani, C. botulinum, C. novyi, C. septicum, and C. chauvoei.

100
Q

Which of these is an adrenergic vasopressor in dogs?

  1. Dopamine
  2. Hydralazine
  3. Glycopyrrolate
  4. Atropine
A
  1. Dopamine

The correct answer is dopamine. Adrenergic vasopressors are frequently used to treat hypotension during anesthesia. Drugs in this category are dopamine, dobutamine, ephedrine, phenylephrine, and norepinephrine. Atropine* and glycopyrrolate are anticholinergic drugs. Hydralazine is a vasodilator used to treat hypertension.

*Atropine increases the heart rate and improves the atrioventricular conduction by blocking the parasympathetic influences on the heart.

101
Q

You are visiting a small “back-yard” flock of chickens and examining the animals. You find several small gray-red mites (approx 0.7 mm) on several of the chickens which you recognize as Dermanyssus gallinae and Ornithonyssus sylviarum. How are these mites best controlled?

  1. Dermanyssus gallinae is controlled primarily by thorough insecticidal treatment of the environment and Ornithonyssus sylviarum must be controlled by application of approved pesticides to affected birds
  2. Control of both Dermanyssus gallinae and Ornithonyssus sylviarum relies on thorough insecticidal treatment of the environment
  3. Control of both Ornithonyssus sylviarum and Dermanyssus gallinae relies on application of approved pesticides to affected birds
  4. Ornithonyssus sylviarum is controlled primarily by thorough insecticidal treatment of the environment and Dermanyssus gallinae must be controlled by application of approved pesticides to affected birds
A
  1. insecticidal treatment of the environment and Ornithonyssus sylviarum must be controlled by application of approved pesticides to affected birds

Dermanyssus gallinae (the common red mite) maintains the majority of the population in the environment so it is important to monitor and treat the environment including the cracks and crevices where mites tend to reside. Ornithonyssus sylviarum mites (northern fowl mites) spend their entire life cycle on the bird. This allows them to multiply more rapidly and makes it essential to treat the birds themselves to control the problem.

102
Q

A 6-year old female spayed English Pointer presents for lethargy and weight gain. The owner notes that the dog is eating and drinking a normal amount, but the dog is still gaining weight. A physical exam reveals weak pelvic limbs, facial nerve paralysis, a symmetrically patchy haircoat, and seborrhea. Lab work reveals a normocytic, normochromic anemia with a PCV of 29% (35-57%), lipemic serum, and cholesterol of 1090 mg/dl (135-278 mg/dl). What is the most likely diagnosis?

  1. Pituitary dependent hyperadrenocorticism
  2. Hypothyroidism
  3. Adrenal dependent hyperadrenocorticism
  4. Hyperthyroidism
A

Hypothyroïdisme

The correct answer is hypothyroidism. The clinical signs, physical exam findings, and lab work abnormalities are classical for hypothyroidism. Other common abnormalities seen with hypothyroidism include pyoderma, neuromuscular signs (ataxia, knuckling, vestibular signs, etc), markedly elevated triglycerides, and a mild normocytic, normochromic anemia. Hyperthyroidism rarely occurs in dogs. You would expect to see polyuria, polydipsia, and polyphagia with hyperadrenocorticism.

103
Q

A 7-year old female Lhasa Apso presents to you for lethargy and inappetence. On your exam, you detect mandibular lymphadenopathy and perform a fine needle aspirate. You see the aspirate depicted here. Which of these treatments would be given to this patient as part of a first line therapy?

  1. Doxycycline
  2. Carboplatin
  3. Prednisone
  4. Milbemycin
  5. Itraconazole
A
  1. Prednisone

*This is a case of lymphoma. The cytology depicts the classic finding of a population of lymphoid cells that are predominantly lymphoblasts based on their size and characteristics. If you were unsure about their size due to magnification, there is a neutrophil in the lower left corner for comparison. The lymphoblasts are considerably larger than the neutrophil. Mature lymphocytes would be smaller than a neutrophil. Note that there is a mitotic figure in the middle of the slide.

There are many treatments and protocols for lymphoma and some of the main agents known to have efficacy are prednisone, doxorubicin, cyclophosphamide, vincristine, L-asparaginase, and lomustine. There are many other efficacious chemotherapeutics for lymphoma but carboplatin is not considered a first line treatment for lymphoma in dogs.

The other drugs listed are antifungal (itraconazole), antibiotic (doxycycline) and anti-parasitic (milbemycin).*

104
Q

A 10-year old male castrated cat that you have previously diagnosed with hyperthyroidism presents to you for acute onset of blindness. You perform an ophthalmic exam and note retinal hemorrhage. What diagnostic test should you perform first?

  1. Blood pressure
  2. Total T4 levels
  3. Serum BUN and creatinine
  4. Free T4 levels by equilibrium dialysis
  5. Coagulation times
A
  1. Blood pressure
    * The correct answer is blood pressure. Cats with hyperthyroidism are likely to develop hypertension. If this is severe enough (>180-200 mmHg systolic), they can be at risk for acute retinal detachment or hemorrhage* resulting in blindness. Prompt resolution of the hypertension is critical to prevent further damage to the eye and other organs.*

*moi: hypertension = dommages vasculaires

105
Q

A horse presents to you with a corneal ulcer. You are concerned because it appears to be infected, as shown in this image. You perform cytology and find gram negative rods. What is the most likely organism infecting the corneal ulcer in this horse?

  1. Pasteurella multocida
  2. E. coli
  3. Pseudomonas
  4. Staphylococcus spp.
A
  1. Pseudomonas

The correct answer is Pseudomonas. Pseudomonas is the most common agent causing bacterial keratitis in the horse. The gram negative cytology provides further evidence that it is the likely culprit. Staphylococcus can cause keratitis in horses but is gram positive. E. coli and Pasteurella are not agents that are commonly involved with keratitis in the horse.

106
Q

Which of these nerve blocks is used to examine the eyes of a horse?

  1. Auriculopalpebral nerve block
  2. Trigeminal nerve block
  3. Corneal nerve block
  4. Oculomotor nerve block
A
  1. Auriculopalpébral

The correct answer is auriculopalpebral nerve block. Blocking this branch of cranial nerve VII (facial nerve) disrupts the motor innervation to the orbicularis oculi, which is the muscle that closes the eye. In the horse, this muscle is very strong and can prevent a thorough ocular exam. To perform this block, lidocaine is injected subcutaneously at the c_audal aspect of the zygomatic arch_ where the nerve is palpable. Because this is a motor nerve, the eye will not be able to close as well, but there is no anesthesia of the tissue.

107
Q

A 6-year old male neutered Weimaraner presents for left forelimb lameness. Radiographs are shown below and show a mixed productive and destructive lesion affecting the left distal radius with accompanying soft tissue swelling. The lesion does not cross the joint. What is the most common primary bone tumor in the dog?

  1. Osteosarcoma
  2. Multiple myeloma
  3. Fibrosarcoma
  4. Hemangiosarcoma
  5. Chondrosarcoma
A
  1. Osteosarcoma

The correct answer is osteosarcoma. Chondrosarcoma, fibrosarcoma and hemangiosarcoma can all be primary bone tumors but are much less common in dogs than osteosarcoma.

108
Q

Which of these are used in the minor cross match for blood products when looking for a compatible blood donor for a dog?

  1. Donor red blood cells, donor plasma
  2. Donor red blood cells, recipient plasma
  3. Recipient red blood cells, recipient plasma
  4. Recipient red blood cells, donor plasma
A
  1. Recipient red blood cells, donor plasma

The correct answer is recipient red blood cells, donor plasma. In a minor crossmatch, you are looking to see if the factors in the plasma of the donor are going to react to the recipient’s red blood cells.

109
Q

A 6-month old cat presents for having ptyalism and for being underweight. On physical examination, you note a bright copper color to the cat’s iris bilaterally. What is this suggestive of?

  1. Polycystic kidney disease
  2. Hemolytic anemia
  3. Portal systemic shunt
  4. Toxoplasmosis
A
  1. Portal systemic shunt

The correct answer is a portal-systemic shunt. Ptyalism is a sign commonly seen with PSS in cats but not dogs, and the copper-colored iris is a striking and almost pathognomonic finding in conjunction with other clinical findings. Hemolytic anemia could cause icterus but not the change in iris color. Animals with polycystic kidney disease would not have a copper-colored iris. Toxoplasma can cause ocular signs such as uveitis but would not have a copper iris.

110
Q

An 11-year old female Pomeranian presents to you for coughing and exercise intolerance. On exam: Wt: 9.25 lbs, T: 101.2F (38.4 C), HR: 132 bpm, RR: Panting, mucous membranes are pink. She has mild tracheal sensitivity and a grade III-IV/VI left apical holosystolic murmur and grade II/VI right apical holosystolic murmur. Femoral pulses are strong and synchronous, with a regular rhythm. She has harsh lung sounds bilaterally. On abdominal palpation, you note hepatomegaly. You find bilateral luxating patellas. You take chest radiographs (see image) and decide to treat the dog based on these findings. Which medication plan is most appropriate?

  1. Furosemide and enalapril
  2. Immiticide (Melarsomine)
  3. Clavamox and enrofloxacin
  4. Atropine and a temporary pacemaker
  5. Oxygen and atenolol
A
  1. Furosemide and enalapril
    * The radiograph here shows a severe symmetrical alveolar pattern in the perihilar region extending to the right and left caudal lung lobes. The heart is tall on the lateral view causing dorsal elevation of the trachea.

This, in conjunction with the physical findings, is compatible with congestive heart failure (CHF) secondary to mitral valve regurgitation. The dog also has hepatic congestion evident by enlargement on the radiograph. Treatment for CHF include diuretics (furosemide/Lasix, hydrochlorothiazide, spironolactone), arterial vasodilators (enalapril, benazepril, amlodipine, hydralazine), positive inotropes (pimobendan), and venodilators (nitroglycerine).

Clavamox and enrofloxacin are antibiotics which could be used to treat pneumonia, but that is not this dog’s problem. Immiticide is the treatment for heartworm. Atenolol is a beta blocker and not part of the management of mitral regurgitation or CHF. Atropine or a pacemaker would be indicated for conduction problems.*

moi: MDCVAV chez le chien; ici on débute avec des signes de d’insuffisance du coeur gauche (congestion pulmonaire) mais éventuellement des problèmes de retour veineux / insuffisance du coeur droig

111
Q

A one-month old foal develops fever, icterus, and diarrhea acutely. Bloodwork shows hyperfibrinogenemia, hypoglycemia, and elevated liver enzymes. Which of these conditions is most likely?

  1. Clostridium novyi type B
  2. Herpesviral hepatitis
  3. Theiler’s disease
  4. Tyzzer’s disease
A
  1. Tyzzer’s disease

The correct answer is Tyzzer’s disease. This is the most likely cause because of the age of the foal and the acute nature of the disease. Tyzzer’s disease is caused by Clostridium piliformis, which causes an acute necrotizing hepatitis. It affects only foals from about 1-6 weeks of age. Theiler’s disease is a condition of adult horses. Clostridium novyi is rare in horses and seen much more in sheep and cattle. Herpesvirus can cause hepatitis but is usually seen at or very soon after birth

112
Q

You are called to examine and treat a valuable 3 year old show cow that appeared normal yesterday but has collapsed and is too weak to get up (see photo). You examine her and find T=102F or 38.9 C, HR=130, and RR=42. Her heart is pounding very loudly. The mucous membranes of her eyes, mouth and vulva are all very pale. You diagnose an acutely bleeding abomasal ulcer. What is the most important treatment?

  1. 40 liters of saline IV
  2. 4 liters of commercial plasma IV plus vitamin K
  3. 4 to 8 liters of fresh whole blood IV
  4. 20 liters of 1.3% sodium bicarbonate IV
  5. 40 liters balanced electrolytes orally
A

4 to 8 liters of fresh whole blood IV

In the case of an acute blood loss such as this, the most important treatment is whole blood. Other s_odium-containing fluids may be beneficial while the blood is being collected_ if this will not result in losing time in getting the whole blood into this cow. In most cases the single blood transfusion results in recovery. After about 24 hours the cow will exhibit melena as the digested blood reaches the rectum.

113
Q

You have a 6-kg cat that you wish to raise his PCV from 15 to 25%. You plan to administer packed red blood cells. How many milliliters of packed red blood cells will this cat need?

  1. 60 ml
  2. 120 ml
  3. 30 ml
  4. 15 ml
A
  1. 60 ml

The correct answer is 60 ml. In order to raise the PCV 1% you will need to give 1ml/kg of packed red blood cells. So it takes 6 mls to raise this cat’s PCV by 1%. If we are going to increase it by 10% we will need 60ml of packed red blood cells

114
Q

Which infectious agent causes the clinical presentation of pigs most similar to that of Haemophilus parasuis (Glasser’s disease)?

  1. Streptococcus suis
  2. Erysipelothrix rhusiopathiae
  3. Fusobacterium necrophorum
  4. Mycoplasma hyosynoviae
A

Streptococcus suis

The correct answer is Streptococcus suis. Streptococcus suis and H. parasuis both cause polyarthritis, polyserositis, fever, and pneumonia in young piglets up to several weeks of age. Both can cause fibrinopurulent inflammation as well as meningitis and convulsions.

Erysipelas and M. hyosynoviae typically occur in grower and finisher pigs and do not result in pneumonia. Diamond-shaped skin lesions (thus the name “diamond skin disease”) are pathognomonic for Erysipelas. Fusobacterium necrophorum causes lameness via footrot or laminitis.

115
Q

Several 16 to 20 month old Holstein dairy heifers who have been out in pasture have developed large areas of skin sloughing, which appears to affect mainly the white unpigmented areas (see photo). Based on this observation, what is the best diagnosis?

  1. Ordinary sunburn
  2. Allergic dermatitis
  3. Photosensitization
  4. Insect hypersensitivity
  5. Malignant catarrhal fever, skin form
A
  1. Photosensitization

Photosensitization mainly affects unpigmented skin where photodynamic agents have accumulated making the skin hyper-reactive to UV light. The cause of this photosensitization can be primary, in which case a plant-derived compound (such as hypericum) or chemical is injected, ingested or topically applied, is the cause. Alternatively, the cause can be secondary due to hepatic damage/failure where the l_iver fails to remove ingested chlorophyll-breakdown products like phylloerythrin, which accumulates in the skin_ and results in UV damage (sunburn). You need to determine whether this is primary or secondary by c_hecking liver enzymes and bilirubin levels_.

116
Q

Which of these drugs is a neuromuscular blocker when given to a dog?

  1. Fentanyl
  2. Naloxone
  3. Neostigmine
  4. Edrophonium
  5. Atracurium
A
  1. Atracurium

The correct answer is atracurium. Atracurium is a non-depolarizing neuromuscular blocker. Other neuromuscular blockers include pancuronium, d-tubocurarine and succinylcholine. Succinylcholine is a depolarizing neuromuscular blocker. Fentanyl is an opioid. Naloxone is an opioid antagonist. Edrophonium and neostigmine are cholinesterase inhibitors used to reverse neuromuscular blockers.

117
Q

What species is the principle carrier and vector of Malignant Catarrhal Fever (MCF) virus in North America?

  1. Cattle
  2. Bison
  3. Sheep
  4. Horse
  5. Donkey
A
  1. Sheep

MCF virus is ovine herpes virus-2 (OvHV-2) and is carried by 95-99% of sheep in North America which show no symptoms. It is also carried by 75% of domestic goats, 40% of muskox, 37% of bighorn sheep, 25% of pronghorn antelope, 62% of mouflon sheep, and by a small percentage of elk, mule deer, and white tailed deer. Susceptible hosts include cattle, water buffalo, deer, pigs, and bison; bison are the most susceptible.

118
Q

Many turkeys on a poultry farm develop whitish “wart-like” nodules and scabs on the comb, wattles, feet, and vent. Which management intervention would help prevent spread of the disease?

  1. Immediate removal of fecal waste
  2. Tick control
  3. Thoroughly disinfect pens and equipment
  4. Raise the room temperature 5 degrees
  5. Mosquito control
  6. Add antibiotics to the drinking water
A
  1. Mosquitoe control

*The condition described here is the dry form of avian (fowl) pox. This is a relatively slow spreading disease that can be spread by contact or by mosquitoes that may harbor infective virus for greater than a month.

In the dry form of the disease, the main sign is raised, whitish wart-like lesions on unfeathered areas (head, legs, vent, etc.). The lesions heal in about 2 weeks. Unthriftiness, decreased egg production and retarded growth may be seen. Mortality is low with this form of the disease. The wet form mainly involves the oral cavity and upper respiratory tract. Lesions are diphtheritic and can ulcerate or erode mucous membranes. Marked respiratory involvement can lead to mortality.

A diagnosis is usually based on flock history and presence of these lesions. This is a pox virus and there is no specific effective treatment but there is a vaccine. Disease control is best accomplished by preventive vaccine as sanitation alone will not prevent spread of disease. Several vaccines are available and a single application results in permanent immunity.

There are not many tick borne poultry diseases but they may include spirochaetosis and Pasteurella infection.

Raising the temperature 5 degrees may be part of the treatment for infectious bronchitis in chickens. Disinfecting pens +/- quarantine is done for quail bronchitis, aspergillosis, and ulcerative enteritis. Antibiotics in the drinking water are most effective for preventing secondary bacterial infections and for mycoplasma but not preventing spread of the virus.*

119
Q

A 1-year old mixed breed stray dog is presented for extreme pruritis and the lesion shown in the picture. The dog was recently found and adopted off the street and has been itching despite being bathed and treated with flea preventatives. The owner reports that she has been itching and developing rashes on her own body since adopting this dog. Your physical exam shows that the dog has several additional similar lesions on the other legs, chest, and ventral abdomen. What is the most likely diagnosis?

  1. Cutaneous lymphoma
  2. Demodex infection
  3. Squamous cell carcinoma
  4. Sarcoptes infestation
A
  1. Sarcoptes

*The key to this question is that Sarcoptic mange is extremely pruritic and can be transmitted to people, including this dog’s owner. Diagnosis of this disease is usually based on clinical impression and potential for exposure to the mites. A positive skin scraping would be most definitive, but scrapes often come back negative and trial therapy would need to be instituted.

Demodex is not transmittable to people and does not typically cause extreme pruritus like Sarcoptes.

Lymphoma and squamous cell carcinoma are less likely because they typically would occur in older animals and neither are usually particularly itchy.*

120
Q

A 11-year old male castrated Golden Retriever presents for collapse with muffled heart sounds on examination. You ultrasound the heart and obtain the following image; the right atrium (RA) and right ventricle (RV) are labeled. A large mass is seen in the right atrioventricular groove with pericardial effusion. What is the likely diagnosis?

  1. Hemangiosarcoma
  2. Pulmonary adenocarcinoma
  3. Lymphoma
  4. Idiopathic pericardial hemorrhage
  5. Chemodectoma
A
  1. Hemangiosarcoma
    * The correct answer is hemangiosarcoma. Hemangiosarcoma of the heart has a p_redilection for the right auricle_ of dogs, with Golden Retrievers being predisposed. Stabilization of this patient will require pericardiocentesis; prognosis is guarded and may include chemotherapy, pericardiectomy, or rarely auriculectomy. Chemodectoma arises from the ascending aorta and heart base; lymphoma is rare in the heart of dogs and is more commonly metastatic than a single mass. Idiopathic pericardial hemorrhage is incorrect because of the clear evidence of a mass.*

Note: Hemangiosarcoma (primaire) most commonly affects the spleen and heart of golden retrievers, Labrador retrievers and German shepherds.

121
Q

When performing a subtotal colectomy on a feline patient, what blood vessel limits the amount of colon that you are able to remove?

  1. Ileocolic artery
  2. Caudal mesenteric artery
  3. Left colic artery
  4. Pudendal artery
A
  1. Ileocolic artery

The correct answer is ileocolic artery. The site for colonic resection is limited by tension on the ileocolic artery when trying to suture your new end of colon to the rectum. Sometimes the tension is too great and instead of a colocolic anastomosis, an ileocolic anastomosis must be performed. Essentially you are trying to connect a section of ascending colon to the rectum. Now that can be pretty far! Performing an ileocolic anastomosis is not ideal because you eliminate the ileocecal valve, and that may predispose the animal to bacterial overgrowth. The caudal mesenteric artery gives branches to the rectum and descending colon. The left colic artery also feeds the descending colon. The pudendal artery supplies the external genitalia. The ileocolic artery provides blood supply to the ascending and transverse colon.

122
Q

A horse is suspected of having the muscle disease rhabdomyolysis, and you are seeking a laboratory test to help confirm the diagnosis. Of the following, which test would likely be the most helpful?

  1. Ionized phosphate
  2. Creatine kinase (CK)
  3. Gamma glutamyl transferase (GGT)
  4. Anion gap
  5. Sorbitol dehydrogenase (SDH)
A
  1. CK

CK, also called, CPK, is found in muscle cells and is elevated in the serum when muscle damage occurs.

123
Q

A 6-month old feedlot steer, which entered the feedlot 4 weeks ago, has lost weight and is now showing an enlarged left flank as shown in the image below. On exam, you find the left side of the abdomen to be gas-filled under moderate pressure, and the rumen to be otherwise poorly filled and with poor motility. Based on percussion and auscultation, the animal also appears to have chronic bronchopneumonia. What is the most likely cause of the rumen malfunction?

  1. Frothy bloat
  2. Type 3 vagal indigestion
  3. Cecal dilation
  4. Free gas bloat, failure to eructate
  5. Left displaced abomasum
A
  1. Free gas bloat, failure to eructate

This is sometimes called Type 1 vagal indigestion, or free gas bloat. It is frequently associated with swollen mediastinal lymph nodes caused by pneumonia. The signals to or from dorsal rumen receptors, which detect gas pressure and open the cardia, are compromised such that eructation does not occur normally and free gas bloat occurs. This in turn causes the calf to feel full and it eats poorly and loses weight. One needs to treat the pneumonia and perhaps create a temporary rumen fistula to allow the escape of gas until eructation returns to normal.

124
Q

A 4-year old male neutered Pit Bull mix comes in to your clinic because his owner thinks he might have gotten into rodenticide while she was out of the house 2 hours ago but is not sure. You send her to bring back the box of rodenticide and induce emesis by administering subconjunctival apomorphine. Within 5 minutes, the dog vomits the material shown in the image below. You rinse out the conjunctiva and administer oral activated charcoal. The owner returns with a box of rodenticide that says brodifacoum. What should you recommend?

  1. Treat the dog with vitamin E and selenium
  2. Treat the dog with vitamin K1 for 6 weeks
  3. No additional treatment or monitoring is needed
  4. Hospitalize the dog for 24 hours to monitor and treat potential neurologic signs
  5. Check serum calcium levels today and once weekly for 6 weeks
A
  1. Treat the dog with vitamin K1 for 6 weeks
    * The bright green vomit confirms the owner’s suspicion of rodenticide ingestion because many rat poisons contain a bright green dye. Dogs that ingest these products may have bright green vomit or stool. Brodifacoum is a vitamin K antagonist commonly used in rodenticides. Ingestion of this compound causes hemorrhaging after several days due to a lack of production of new clotting factors. Treatment for this condition requires vitamin K1 administration for 4-6 weeks.*
    note: hémorragies 3-5 jours post-exposition suite à la déplétion des facteurs de coagulation 2,7,9,10.

vitK1 = phylloquinone

125
Q

A 4-year old female spayed mixed Chihuahua presented to the emergency service at approximately 5am this morning after presumptively being attacked by a coyote. The patient has a flail chest and it is questionable if there is direct communication between the thoracic cavity and the environment (it was difficult to examine the dog due to her fractious nature). Exploration of the wound was performed, and once anesthetized, it was apparent she had a pneumothorax. The patient must be ventilated, as there is no vacuum present in the chest for lung expansion to occur. What pressure should the anesthetist not exceed if manually bagging the patient during anesthesia

  1. 8cm H2O
  2. 24cm H2O
  3. 12cm H2O
  4. 20cm H2O
A

20cm H2O

Pressures above 20cm H20 may result in barotrauma. In an otherwise healthy patient it is not recommended to exceed this pressure. In patients with chronic atelectasis, anesthetists will be much more apprehensive about over ventilating or ventilating the lungs too quickly, as acute expansion can trigger re-expansion pulmonary edema, which may then lead to acute respiratory distress syndrome and death.

126
Q

Which of these is the most common cause of seizures in the adult ferret?

  1. Idiopathic epilepsy
  2. Hypocalcemia
  3. Hypoglycemia
  4. Intracranial neoplasia
  5. Hepatic encephalopathy
A
  1. Hypoglycemia

Hypoglycemia secondary to insulinoma is the most common cause of seizures in the adult ferret. Keep in mind, a prolonged seizure can actually cause hypoglycemia, so the finding of low blood glucose in a seizuring ferret does not necessarily confirm a diagnosis of insulinoma. Idiopathic epilepsy has not been reported in ferrets. Hypocalcemia and hepatic encephalopathy can cause seizures, but are not as commonly reported as hypoglycemia.

127
Q

A male goat presents for vocalizing and straining to urinate. He has been kicking at his abdomen. You detect crystals adherent to the hairs around the prepuce (see image). What condition should you suspect and try to rule out first?

  1. Urinary tract obstruction
  2. Lower intestinal obstruction
  3. Acute severe pyelonephritis
  4. Upper intestinal obstruction
A
  1. Urinary tract obstruction

The correct answer is urinary tract obstruction due to calculi. This should be suspected in all male and castrated male goats (and sheep) with non-specific signs of disease or discomfort because it is so common. Common clinical signs associated with urinary tract obstruction are vocalization and dribbling of urine. Heaving or forceful abdominal contractions may be seen. Hematuria, dysuria, prolonged urination, and apparent abdominal pain are also common signs. The most important step of evaluation is exteriorization of the penis and examination of the urethral process because this is the most common site of blockage. In severe cases the entire urethra may be filled with calculi.

128
Q

Rubiosis iridis as seen in this cat is a sign of what process in the eye?

  1. Lens luxation
  2. Chorioretinitis
  3. Iris atrophy
  4. Anterior uveitis
  5. Glaucoma
A
  1. Anterior uveitits
    * The correct answer is anterior uveitis. Rubiosis iridis along with other signs such as aqueous flare, hyphema, hypopyon, keratic precipitates, and decreased intraocular pressure are all suggestive of anterior uveitis which can be caused by a number of infectious, immune-mediated, traumatic, and idiopathic causes.*

Note: rubiosis iridis: abnormal blood vessels (formed by neovascularization) are found on the surface of the iris

129
Q

A 2-year old female DSH cat presents for weight loss, anorexia, dyspnea, and lethargy. She was previously treated with antibiotics but is still febrile on physical exam. You detect pleural effusion and notice that the abdomen is distended. On CBC there is a non-regenerative anemia, neutrophilia, and lymphopenia. On chemistry there is hyperproteinemia and a slight elevation in liver enzymes. What is your primary differential?

  1. Feline immunodeficiency virus
  2. Feline calicivirus
  3. Feline infectious peritonitis
  4. Feline leukemia virus
A
  1. Feline Infectious Peritonitis

The correct answer is FIP. This poor cat is infected with the dreaded wet form of feline infectious peritonitis.

If you think about the pathogenesis of the virus, then these clinical signs make a lot of sense. With FIP, the classical lesion is pyogranulomatous vasculitis due to antigen-antibody complexes depositing in the venular endothelium, which results in pleural and peritoneal effusion. The CBC findings are common for FIP but not too specific. In greater than 50% of cats with the wet form of FIP, there will be hyperproteinemia. In greater than 70% of cats with the dry form of FIP there is hyperproteinemia.

130
Q

During your examination of an 18-year old horse, you observe what is shown in the photograph. Based on the location and appearance of this lesion, what is the most likely diagnosis?

  1. Habronema
  2. Sarcoid
  3. Cuterebra
  4. Melanoma
  5. Squamous cell carcinoma
A
  1. Melanoma

The correct answer is melanoma. Melanoma is one of the most common tumors in horses (about 10% of all neoplasms) and gray horses are at high risk of around 80%. They can occur anywhere but appear most frequently in the perineal region or ventral tail as is evident in this horse.

In horses, they are usually darkly pigmented (as opposed to dogs where amelanotic melanomas occur somewhat commonly). In horses, most are slowly growing but can be locally invasive. Many treatments are out there, but there is no standard of care treatment. Depending on location and extent, consider surgical removal, benign neglect, chemotherapy (systemic or intralesional) and immunotherapy.

131
Q

A 10-day old commercial dairy calf has diarrhea that is white in color (see photo). The calf is dehydrated, hypovolemic, weak and unable to stand. T=100F (37.8 C), HR=100, and RR=20. No other abnormalities are found on physical examination. Based on these findings, what is the treatment of choice?

  1. IV fluids with added sodium bicarbonate
  2. IV fluids containing 50 meq/L of potassium
  3. Oral fluids containing sodium bicarbonate
  4. Oral fluids containing high levels of both sodium and chloride
  5. IV fluids containing only saline
A
  1. IV fluids with added sodium bicarbonate

This calf is typical of those suffering from nonspecific calf diarrhea, most often associated with enteropathogenic E coli, rotavirus, or cryptosporidium. The calf develops hypovolemia and metabolic acidosis and requires sodium-containing IV fluids which contain additional alkali such as sodium bicarbonate. One can assess that the calf is severely acidotic given that it is lethargic and unable to stand. While oral fluids may also be useful, at this stage, the calf will require IV fluids.

132
Q

A horse presents to you for chronic, recurrent laminitis and skin disease. You notice on your exam that the horse has a particularly thick, long, wavy, and matted coat. The owner mentioned that this developed many months ago. What is a likely diagnosis?

  1. Pheochromocytoma
  2. Diabetes insipidus
  3. Hypothyroidism
  4. Cushing’s disease (Pituitary Pars Intermedia Dysfunction)
A

The correct answer is Cushing’s disease. The coat condition described is what horses with glucocorticoid excess develop; it is referred to as hirsutism. They will also be predisposed to infections including laminitis and skin diseases such as Dermatophilus. They are also frequently polyuric, polydipsic, and polyphagic.

133
Q

A 9-year old male Queensland Heeler presents with a four day history of progressive tetraparesis. Physical exam showed him to be weakly ambulatory with support. As part of your initial workup, you take chest X-rays which are shown below. Which of the following next steps is the most appropriate test to confirm your clinical suspicion about the cause of the dog’s signs?

  1. Myelogram
  2. MRI of the brain
  3. CT scan of the thorax
  4. Tensilon response test
  5. Bronchoalveolar lavage and culture
A
  1. Tensilon response test

The correct answer is a Tensilon (edrophonium) response test. Hopefully, you were able to identify the mass in the cranial mediastinum on the chest radiograph, as this was one of the keys to this case. This, in conjunction with the dog’s other signs, are suggestive that this dog has a thymoma and associated secondary myasthenia gravis. Tensilon (edrophonium) is a rapidly acting anticholinesterase that reverses signs of myasthenia within minutes in most dogs.

A chest CT would be a valid test to confirm the presence of the mediastinal mass and might be an appropriate test before surgery but would not bring you closer to a diagnosis if you already have identified the mass. An MRI of the brain would assess a CNS cause of the dog’s signs, which are unlikely, given the other findings. Similarly, a myelogram would assess if a spinal cord lesion caused the dog’s signs, but the rest of the findings in this case should point you in a different direction.

134
Q
A
135
Q

You are asked to examine some feeder pigs that have stopped eating yesterday. The group is lying down and seems lethargic. They have fevers of 105-106F (40.6 -41.1 C), firm dry feces, and the skin has rhomboid-shaped red blotches scattered on it. What treatment should be recommended?

  1. Penicillin
  2. Metronidazole
  3. Chloramphenicol
  4. Gentamicin
  5. Streptomycin
A

Erysipelas is susceptible to penicillins, as well as tetracyclines (usually), lincomycin and tylosin. Chloramphenicol and nitroimidazoles (including metronidazole) are not approved for food animal use.

136
Q

What is the potential udder fate of goats infected with caprine arthritis encephalomyelitis virus?

  1. Hardbag
  2. The udder is not affected
  3. Bluebag
  4. Gangrene
A
  1. Hardbag

CAEV is a retrovirus and when it affects the udder it will cause fibrosis and result in a firm udder with agalactia. Treatment is ineffective and the goat should be culled. The disease is usually subclinical but can cause arthritis in adults and encephalitis in kids.

137
Q
A